You are on page 1of 83

Marian Mureşan

A Concrete Approach to
Classical Analysis

Solutions to Exercises
February 29, 2012

Springer
Contents

1 Sets and Numbers . . . . . . . . . . . . . . . . . . . . . . . . . . . . . . . . . . . . . . . . . 5

2 Vector Spaces and Metric Spaces . . . . . . . . . . . . . . . . . . . . . . . . . . 25

3 Sequences and Series . . . . . . . . . . . . . . . . . . . . . . . . . . . . . . . . . . . . . . 27

4 Limits and Continuity . . . . . . . . . . . . . . . . . . . . . . . . . . . . . . . . . . . . . 39

5 Differential Calculus on R . . . . . . . . . . . . . . . . . . . . . . . . . . . . . . . . . 49

6 Integral Calculus on R . . . . . . . . . . . . . . . . . . . . . . . . . . . . . . . . . . . . 57

7 Differential Calculus on Rn . . . . . . . . . . . . . . . . . . . . . . . . . . . . . . . . 69

8 Double Integrals, Triple Integrals, and Line Integrals . . . . . . 75


1
Sets and Numbers

1.1. (i) Suppose that the set in the left-side contains an element, say x. Then
x belongs to A, not to B, and to B. We get a contradiction, hence the set
in the left-side contains no element.
(ii) Obviously, A ∩ B ⊂ A \ (A \ B). If there exists an x ∈ A \ (A \ B), then
x ∈ A and x ∈ / A \ B. It follows that x ∈ B, so that x ∈ A ∩ B.
(iii) Both sides of this identity contain elements belonging either to A or to
B.
(iv) Both sides of the identity contain elements in A and neither in B nor in
C.
(v) The three members of this double identity contain elements belonging to
A and C, but not to B.

1.2. Suppose that A \ B ⊂ C and pick up an x ∈ A. If x ∈ / B, then


x ∈ A \ B and thus x ∈ C. If x ∈ B, then x ∈ B ∪ C. Conversely, suppose
that A ⊂ B ∪ C and pick up an x ∈ A \ B. Then x ∈ A and x ∈ / B. Thus
x ∈ (B ∪ C) \ B ⊂ C.

1.3. (i) We have

x ∈ (A ∪ B) \ C ⇐⇒ (x ∈ A or x ∈ B) and x ∈/C
⇐⇒ (x ∈ A and x ∈/ C) or (x ∈ B and x ∈
/ C)
⇐⇒ x ∈ (A \ C) ∪ (B \ C).

(ii) We have

x ∈ (A ∩ B) \ C ⇐⇒ (x ∈ A and x ∈ B) and x ∈
/C
⇐⇒ (x ∈ A and x ∈ / C) and (x ∈ B and x ∈
/ C)
⇐⇒ x ∈ (A \ C) ∩ (B \ C).
6 1 Sets and Numbers

(iii) By (i) we have

(A ∪ C) \ B = (A \ B) ∪ (C \ B) ⊂ (A \ B) ∪ C.

1.4. (i) One can easily show that Y = (B \ C) \ A and Y ⊂ X. If C = ∅ and


A = B ̸= ∅, then ∅ = Y ̸= A = X. We conclude that Y ⊂ X.

1.5. (i)

x ∈ ∪∞
n=0 Bn ⇐⇒ ∃n ∈ N, x ∈ Bn ⇐⇒ ∃n ∈ N, i ∈ {0, . . . , n}, x ∈ Ai
⇐⇒ x ∈ ∪∞
n=0 An .

(ii)

x ∈ ∩∞
n=0 Bn ⇐⇒ ∀n ∈ N, x ∈ Bn ⇐⇒ ∀n ∈ N, i ∈ {0, . . . , n}, x ∈ Ai
⇐⇒ x ∈ ∩∞
n=0 An .

1.6. (i)

x ∈ ∪∞ ∞ ∞
m=0 (∩n=0 Am,n ) =⇒ ∃m0 ∈ N, x ∈ ∩n=0 Am0 ,n
=⇒ ∃m0 ∈ N, ∀n ∈ N, x ∈ Am0 ,n
=⇒ x ∈ ∩∞ ∞ ∞ ∞
n=0 (∪m=m0 Am,n ) ⊂ ∩n=0 (∪m=0 Am,n ).

(ii) Not true.

1.7. (i) {X ∅ = X \ ∅ = X and {X X = X \ X = ∅.


(ii) {(A \ B) = {(A ∩ {B)) = ({A) ∩ ({({B)) = ({A) ∪ B.
(iii)

(A ∪ B) \ (A ∩ B) = (A ∪ B) ∩ ({(A ∩ B)) = (A ∪ B) ∩ ({A ∪ {B)


= (A ∩ ({A ∪ {B)) ∪ (B ∩ ({A ∪ {B))
= ((A ∩ {A) ∪ (A ∩ {B) ∪ ((B ∩ {A) ∪ (B ∩ {B) = (A ∩ {B) ∪ ((B ∩ {A).

(iv) x ∈ {B =⇒ x ∈
/ B =⇒ x ∈
/ A =⇒ x ∈ {A.

1.8. Successively we have

{(X ∪ A) ∪ (X ∪ {A) = B ⇐⇒ (({X ∩ {A) ∪ {A) ∪ X = B


⇐⇒ (({X ∪ {A) ∩ ({A ∪ {A)) ∪ X = B ⇐⇒ (({X ∪ {A) ∩ {A) ∪ X = B
⇐⇒ {A ∪ X = B ⇐⇒ A ∩ ({A ∪ X) = A ∩ B ⇐⇒ A ∩ X = A ∩ B.

Thus X is any subset of U satisfying A ∩ X = A ∩ B.


1.1 Solutions 7

1.9. (i)

X ∈ P(A) ∪ P(B) =⇒ X ∈ P(A) or X ∈ P(B)


=⇒ X ⊂ A or X ⊂ B =⇒ X ⊂ A ∪ B =⇒ X ∈ P(A ∪ B).

(ii) Pick a nonempty set X ∈ P(A ∪ B). Then X ⊂ A ∩ B. If X ⊂ A or


X ⊂ B, the equality is true. Suppose that X = X1 ∪ X2 , with X1 ̸= ∅,
X1 ⊂ A, X2 ̸= ∅, X2 ⊂ B, and X1 ̸= X2 . Then X ∈
/ P(A) ∪ P(B).
(iii) and (iv) are straightforward.

1.10. (i) One-to-one.


(ii) Onto.
(iii) One-to-one and onto.

1.11. (a) (i) =⇒ (ii) Consider f∗ (M1 ) = f∗ (M2 ). This is equivalent to


f (M1 ) = f (M2 ). We show that M1 = M2 . Pick an arbitrary x ∈ M1 . Then
f (x) ∈ f (M1 ) = f (M2 ). It means that there exists y ∈ M2 with f (x) = f (y).
Since the function f is one-to-one, it follows that x = y and x ∈ M2 . Thus
M1 ⊂ M2 . In a similar way we show that M2 ⊂ M1 , and therefore M1 = M2 .
Thus the function f∗ is one-to-one.
(ii) =⇒ (iii) Let M ∈ P(A) be arbitrary. We show that there exists N ∈
P(B) with f ∗ (N ) = M, that is, for M ⊂ A there exists N ⊂ B with
f −1 (N ) = M. Define N = f (M ). From the injectivity of f we indeed have
that M = f −1 (N ) = f ∗ (N ).
(iii) =⇒ (iv) The inclusion f (M ∩ N ) ⊂ f (M ) ∩ f (N ) is obvious. We now
show its reverse. Let N1 , N2 ∈ P(B) be with f −1 (N1 ) = M1 and f −1 (N2 ) =
M2 be arbitrary. We have f −1 (N1 ∩ N2 ) = f −1 (N1 ) ∩ f −1 (N2 ). For y ∈
f (M1 ) ∩ f (M2 ) there exists x1 ∈ M1 , x2 ∈ M2 with f (x1 ) = f (x2 ) = y.
Since f (x1 ) ∈ N1 , f (x2 ) ∈ N2 , it follows that y ∈ N1 ∩ N2 . Thus f −1 (y) ∈
f −1 (N1 ∩N2 ) = f −1 (N1 )∩f −1 (N2 ) = M1 ∩M2 , and there exists x ∈ M1 ∩M2
with f (x) = y. So f (M ) ∩ f (N ) ⊂ f (M ∩ N ).
(iv) =⇒ (v) Taking M1 = M and M2 = {M, we have that f (M ) ∩ f ({M ) =
f (M ∩ {M ) = f (∅) = ∅ and thus f ({M ) ⊂ {f (M ).
(v) =⇒ (i) Consider x, y ∈ A with f (x) = f (y) and define M = A \ {x}.
Then {M = {x} and f (x) = f (y) ∈ f ({M ) ⊂ {f (M ). Thus f (y) ∈ / f (M )
and y ∈ / M. Therefore y = x and the function f is one-to-one.
(b) The proof is similar to (a).
(c) Follows from (a) and (b).

1.12. (a) (i) =⇒ (ii) For every ξ ∈ C we have that (f ◦ g)(ξ) = (f ◦ h)(ξ),
i.e., f (g(ξ)) = f (h(ξ)). Because of f is one-to-one, it follows that g(ξ) = h(ξ)
and since ξ was taken arbitrarily in C, we have that g = h.
(ii) =⇒ (i) Suppose that f is not one-to-one. Then there exist x, y ∈ A, with
x ̸= y and f (x) = f (y). Consider C = {u, v} and the functions g, h : C → A
8 1 Sets and Numbers

satisfying g(u) = x, g(v) = y, and h(u) = h(v) = y. Then we have f ◦g = f ◦g


and g ̸= h.
(b) (i) =⇒ (ii) Let y ∈ B be arbitrary. Since f is onto, there exists x ∈ A so
that f (x) = y. We have the sequence of implications g(f (x)) = h(f (x)) =⇒
g(y) = h(y) =⇒ g = h.
(ii) =⇒ (i) Suppose f is not onto. Then there exists y ∈ B so that for every
x ∈ A, f (x) ̸= y. Consider C = {0, 1} and g(z) = 0, for all z ∈ B, h(z) = 0,
for all z ∈ B \ {y} and h(y) = 1. Then g ◦ f = h ◦ f and g ̸= h.

1.13. (i) There exists an injection N∗ ∋ n → 2n ∈ N∗ .


(ii) The set is at least countable since the coordinates a given vertice runs
on the countable set Q × Q. The same set is a subset of the countable set
(Q × Q) × (Q × Q) × (Q × Q), Theorem 2.17.
(iii) The set coincides with Q × Q which is countable.
(iv) It follows from Theorem 2.17.

1.14. (i) Denote N = {a1 , a2 , . . . , an } and M = {b1 , b2 , . . . , bm } Then to a1


one can assign an arbitrary element from M, i.e., there are m possibilities, to
a2 one can assign an arbitrary element from M, i.e., there are m possibilities,
and so on.
(ii) Follows from (iii).
(iii) To a1 one can assign an arbitrary element from M, let it be {bi }, i.e.,
there are m possibilities, to a2 one can assign an arbitrary element from
M \ {bi }, let it be {bj }, i.e., there are m − 1 possibilities, to a3 one can
assign an arbitrary element from M \ {bi , bj }, let it be {bk }, i.e., there are
m − 2 possibilities, and so on till to each element from M is assigned one
from N.
(iv) Denote the set of onto functions from M into N by Sm n
, m ≥ n. For
each i = 1, . . . , n, let Ai be the set of functions from M into N such that
n
bi does not belongs to the image of any such function. Then the set Sm
of onto functions from M into N coincides with the set of all functions
from M into N except the functions belonging to some of the sets Ai . Then
|Smn
| = nm − |A1 ∪ A2 ∪ · · · ∪ An |. By the first equality in Exercise 1.18, we
expand |A1 ∪ A2 ∪ · · · ∪ An |, and remark that

|Ai | = (n − 1)m ,|Ai ∩ Aj | = (n − 2)m , . . . , ∩ni=1 Ai = ∅.


( ) ( )
We also note that there are n1 sets Ai , n2 sets Ai ∩ Aj , . . . . Now the result
follows.

1.15. First approach. One proves them by induction.


Second approach. One proves them by combinatorial argues. For the first
identity a combinatorial argue is the next one. Choose a set A of n ∈ N
elements and a set B with two elements 0 and 1. We identify a subset M ⊂ A
with a mapping f : M → B defined as f (x) = 1 if x ∈ M and f (x) = 0
1.1 Solutions 9

otherwise. Thus we obtain a bijection between the family of subsets of A


and the set of functions from M into B. The cardinal of the later set is 2n ,
according to (i) in Exercise 1.14. For the next two identities consider C and
D a partition of family of subsets of A so that C contains subsets of odd
cardinal whereas D contains subsets of even cardinals. We already saw that
|C ∪ D| = 2n . Pick up an element in A, denote it x. We define a bijection f
from C onto D such that f (X) = X ∪ {x} if x ∈ / X and f (X) = X \ {x}
otherwise. It follows that the sets C and D are of the same cardinal and thus
the last two identities are proved.
One can try using the next Mathematica r commands.
TraditionalForm[Sum[Binomial[n, k], {k, 0, n},Assumptions
\[RightArrow] n\[Element]Integers &&n\[GreaterSlantEqual] 0]]
TraditionalForm[Sum[Binomial[n,2*k],{k,0,n/2},Assumptions
\[RightArrow] n\[Element]Integers &&n\[GreaterSlantEqual] 0]]
TraditionalForm[Sum[Binomial[n,2*k+1],{k,0,n/2},Assumptions
\[RightArrow] n\[Element]Integers &&n\[GreaterSlantEqual] 0]]

1.16. Let A be the family of partitions of S. Then

S = {1} =⇒ A = {S},
S = {1, 2} =⇒ A = {S, {{1}, {2}}},
S = {1, 2, 3} =⇒ A = {S, {{1}, {2}, {3}}, {{1, 2}, {3}}, {{1, 3}, {2}},
{{2, 3}, {1}} },
S = {1, 2, 3, 4} =⇒ A = {S, {{1}, {2}, {3}, {4}}, {{1, 2, 3}, {4}},
{{1, 2, 4}, {3}}, {{1, 3, 4}, {2}}, {{2, 3, 4}, {1}}, {{1, 2}, {3, 4}},
{{1, 2}, {3}, {4}}, {{1, 3}, {2, 4}}, {{1, 3}, {2}, {4}}, {{1, 4}, {2, 3}},
{{1, 4}, {2}, {3}}, {{2, 3}, {1}, {4}}, {{2, 4}, {1, 3}}, {{3, 4}, {1}, {2}}}.

Thus the equalities in (1.11) of the statement hold. Some results can be ob-
tained by Mathematica r , Figure 1.1.
One can also try using the next Mathematica r command.
Table[BellB[n], {n, 0, 4}]
Now suppose S = {a1 , a2 , . . . , an , an+1 }. Pick up( an
) arbitrary element in S,
say an+1 . Select k elements in S \ {an+1 } (in nk ways) and denote by A
the set of remaining n − k elements. By A one has Bn−k partitions. Thus we
get (1.12).

1.17. Every x ∈ A belongs to at least a set Ai . The set of indices of the sets
Ai containing x is a nonempty subset to {1, 2, . . . , n}. So, there are 2k − 1
possibilities for x belonging to at least a set Ai . Since each x ∈ A belongs to
at least a set Ai , the conclusion follows.
10 1 Sets and Numbers

H* Set partitions*L
SetPartitions@4D
8881, 2, 3, 4<<, 881<, 82, 3, 4<<, 881, 2<, 83, 4<<, 881, 3, 4<, 82<<,
881, 2, 3<, 84<<, 881, 4<, 82, 3<<, 881, 2, 4<, 83<<, 881, 3<, 82, 4<<,
881<, 82<, 83, 4<<, 881<, 82, 3<, 84<<, 881<, 82, 4<, 83<<, 881, 2<, 83<, 84<<,
881, 3<, 82<, 84<<, 881, 4<, 82<, 83<<, 881<, 82<, 83<, 84<<<

H* Bell numbers*L

BellB@4D
15

Fig. 1.1. Set partitions and Bell numbers for n = 4

1.18. By induction.

1.19. Both sides increase with ⌈y⌉, if x is substituted by x + 1. Therefore


we can consider that 0 ≤ x < 1. Both sides vanish for x = 0 and both sides
increase by 1 if x increases over 1 − k/y, for 0 ≤ k < y.
⌊ ⌋
n + 2k
1.20. There exists k0 ∈ N∗ such that n < 2k0 . Then = 0 for all
2k+1
k ≥ k0 . Thus the left-hand side of (1.13) in the statement contains precisely
a finite sum.
For any real x we have
⌊ ⌋
1
x+ = ⌊2x⌋ − ⌊x⌋.
2

Applying the above relation to each nonzero term in the left-side of (1.13),
we get the result.

1.21. Suppose (a − r)n + an = (a + t)n . Denote t = a/r. Then (t − 1)n + tn =


(t + 1)n , that is
(( ) ( ) )
n n−1 n n−3
tn − 2 t + t + · · · + 1 = 0.
1 3

Since the coefficient of the leading term is 1 and all the other coefficients are
integers, the rational roots are integers. If t is odd, we get a contradiction.
If t is even, the leading term is divisible by 4, while the product is divisible
only by 2. We get a contradiction again. Thus the polynomial equation has
no integer root. So it has no rational root. ⊓ ⊔

1.22. Suppose that |A| = n. Then P(A) is equivalent with the set of n-term
sequences of 0 and 1.
1.1 Solutions 11

1.23. Multiply (1.14) by x and add −1. Then (1.14) is equivalent to
√ √ √ √
−( x + 1 − x)2 < 0 < ( x − x − 1)2 ,

which are obvious. Now we return.

1.24. Since the inequality is symmetric in the variables, we may assume that
x ≥ y ≥ z. Then we can write

(x − y)(xr (x − z) − y r (y − z)) + z r (x − z)(y − z) ≥ 0,

and every term on the left hand side is nonnegative.

1.25. By induction.

1.26. For x = −1 the left-hand side of (1.16) in the statement is null while
the right-hand is non-positive. So we may suppose that x > −1. But for x >
−1 inequality (1.16) follows immediately from inequality (1.15) considering
xi = x, i = 1, 2, . . . , n.

1.27. (i) In (1.16) we set x = 1/n to get the left hand side inequality. To get
the other one we apply the Newton formula. If an = (1 + 1/n)n , then
( ) ( ) ( )
n 1 n 1 n 1
an = 1 + + + ··· +
1 n 2 n2 n nn
n(n − 1) 1 n(n − 1)(n − 2) . . . 2 · 1 1
=1+1+ 2
+ ··· + n
( 2! ) n ( ) (n! )( n )
1 1 1 2 n−1 1
=1+1+ 1− + ··· + 1 − 1− 1−
n 2! n n n n!
1 1 1
< 1 + 1 + + + ··· +
2! 3! n!
Since n! ≥ 2n−1 , for every n ≥ 2, it follows that
( )
1 1 1
2 < an < 1 + 1 + + 2 + · · · + n−1 < 3.
2 2 2

(ii) The inequality is equivalent to



n+1
n+1

nn
≤1

or
√ √ √( )n
n+1
n+1 n(n+1) (n + 1)n n(n+1) 1 1

nn
= = 1+ .
nn+1 n n
12 1 Sets and Numbers

By (i) we have that ( )n


1
1+ < 3 ≤ n.
n
Thus
√ √( )n √
n+1
n+1 n(n+1) 1 1 n(n+1) n

nn
= 1+ ≤ = 1.
n n n

1.28. Cauchy’s approach. First we prove the mean inequality introducing an


extra assumption, namely m = 2k . Later on we will remove this assumption.
If k = 1, the mean inequality is known. For k = 2 we follow the next way
√ √ √
√ √ √ x1 x2 + x3 x4
4
x1 x2 x3 x4 = x1 x2 x3 x4 ≤
2
x1 + x2 x3 + x4
+ x1 + x2 + x3 + x4
≤ 2 2 = .
2 4
Now, suppose that inequality (1.17) holds for m = 2k , i.e.,

2√
k x1 + x2 + · · · + x2k
x1 x2 . . . x2k ≤ . (1.1)
2k
Then


2k+1 2√
k √
k
x1 x2 . . . x2k+1 = x1 . . . x2k 2 x2k +1 . . . x2k+1
2√
k √
k
x1 . . . x2k + 2 x2k +1 . . . x2k+1
≤ .
2
Using (1.1), we get that (1.17) holds for any m equal to a power of 2.
It remained the case of unrestricted m. For, suppose that m is not a power
of 2. Take a k ∈ N∗ such that m < 2k and denote
x1 + x2 + · · · + xm
l= (> 0).
m
Set
xm+1 = · · · = x2k = l
and consider inequality (1.1). Then we write

2√ ml + (2k − m)l
x1 x2 . . . xm · l(2 −m)/2 ≤
k k k
=l
2k
2√
k k
x1 x2 . . . xm ≤ lm/2 ⇐⇒ x1 . . . xm ≤ lm .

Second approach. This approach consists in considering a special case of (1.17)


which is, actually, equivalent to it. This special case reads as
1.1 Solutions 13

x1 , . . . , xm > 0, x1 · · · xm = 1 =⇒ x1 + x2 + · · · + xm ≥ m. (1.2)

If m = 1 or m = 2, (1.2) is trivial. Suppose that (1.2) holds for m = n,


i.e.,
x1 , . . . , xn > 0, x1 . . . xn = 1 =⇒ x1 + x2 + · · · + xn ≥ n. (1.3)
Suppose now that there are given n + 1 positive real numbers satisfying

x1 x2 · · · xn xn+1 = 1.

We prove that their sum is at least n + 1.


If either all xi ’s are equal to 1, and then the conclusion follows at once, or
there are two numbers one greater than 1, the other less than 1. We suppose
that xn > 1 and xn+1 < 1. Now we consider n positive numbers, namely

x1 , x2 , . . . , xn−1 , xn · xn+1 .

Based on the induction hypothesis, (1.3), we get the following lower estimate

x1 + x2 + · · · + xn−1 + xn xn+1 ≥ n.

Further

x1 + x2 + · · · + xn+1 ≥ n − xn xn+1 + xn + xn+1


= n + 1 + (1 − xn+1 )(xn − 1) ≥ n + 1.

Thus we infer that implication (1.3) holds for every m ∈ N∗ . Hence the mean
inequality is proved on this the second way as well.

1.29. Substitute xi → 1/xi in (1.17).

1.30. (i) It follows from the proof of Exercise 1.28.


(ii) Multiply the left-hand side of the inequality in Exercise 1.29 with the
right-hand side of the inequality in Exercise 1.30.

1.31. Successively we have


ab
(xi − a)(b − xi ) ≥ 0 =⇒ xi + ≤a+b
xi
ti
=⇒ ti xi + ab ≤ (a + b)ti , ∀ i = 1, 2, . . . , n
xi
and
14 1 Sets and Numbers
∑ ∑ ti ∑
ti xi + ab ≤ (a + b) ti
xi
(∑ ) ( ∑ ti ) ( ∑ ∑ )2 ( ∑ )2
ti xi + ab (ti /xi ) (a + b) ti
=⇒ ti x i ab ≤ ≤
xi 2 2
( n )2
(∑ ) (∑ t ) (a + b)2 ∑
i
=⇒ ti xi ≤ ti .
xi 4ab i=1

1.32. (i) It follows from


n
1 ∑n ∑ 1
n
(n − 1)s = (s − ak ) ≥ n2 .
s − ak s − ak
k=1 k=1 k=1

(ii) Set m = n, xi = i for i ∈ {1, 2, . . . , n − 1}, and xn = n/2 in (1.17).


We can approach inequality (1.19) by induction, too. For n = 1 and n = 2
we get equality. For n = 3 we get 3! < 2(3/2)3 , which is obvious. Suppose
(1.19) holds for an n ∈ N∗ , n ≥ 3. Then using exercise 1.27 for n ≥ 3, we
have
( n )n ( )n+1 ( )n+1
n+1 2 n+1
(n + 1)! < 2 (n + 1) = 2 ≤ 2 .
2 2 (1 + 1/n)n 2

Thus (1.19) is completely proved by induction.

1.33. (i) It follows from the mean inequality.


(ii) It follows from (i) and called the Maclaurin inequality.

1.34. (i) It follows at once by induction.


(ii) It follows from (i) or directly by induction.
(iii) From (i) and (ii) we have that

∑ (√ √ )2
ti /tj − tj /ti < 1.
1≤i<j≤n

Suppose there exists a triple a, b, c not forming a triangle. Then we suppose


a ≥ b + c. We have
(√ √ )2 (√ √ )2
a/b − b/a + a/c − c/a
= (a − b)2 /(ab) + (a − c)2 /(ac) ≥ (a − b)c/(ab) + (a − c)b/(ac)
= c/b + b/c − (b + c)/a ≥ 2 − 1 = 1,

contradicting the previous inequality.


1.1 Solutions 15

1.35. (i) First approach. We just remark that (1.21) follows from Lagrange
identity (1.20).
Second approach. Consider the following inequalities

(ai x + bi )2 = a2i x2 + 2ai bi x + b2i ≥ 0, ∀ x ∈ R, i = 1, 2, . . . , m

and add them. Then


(m ) (m )
∑ ∑ ∑
m
2 2
ai x + 2 ai bi x + b2i ≥ 0, ∀ x ∈ R.
i=1 i=1 i=1

From here (1.21) follows.


(iii) It follows from (1.21).
(iv) Take bi = 1, i = 1, . . . , m, in inequality (1.21).
(v) We have
a2 (a − b)2
= 2a − b +
b b
and the other two similar equalities. Summing them up, we get

a2 b2 c2 (a − b)2 (b − c)2 (c − a)2


+ + =a+b+c+ + + .
b c a b c a
By the Cauchy inequality, we have
( )
(a − b)2 (b − c)2 (c − a)2
(a + b + c) + + ≥ (|a − b| + |b − c| + |c − a|)2
b c a
≥ (2|a − b|)2 .

1.36.

n ∑
n ∑
n ∑
n
a2k ≤ an ak and b2k ≤ bn bk .
k=1 k=1 k=1 k=1

Multiply the two inequalities to get


( n )( n ) ( n )( n )
∑ ∑ ∑ ∑
2
ak bk ≤ an bn
2
ak bk
k=1 k=1 k=1 k=1
( )( )( )

n ∑
n ∑
n
< ak bk ak bk (mean inequality)
k=1 k=1 k=1
( )2 ( )2 ( )2 /

n ∑
n ∑
n
≤ ak bk + ak bk  2
k=1 k=1 k=1
16 1 Sets and Numbers
( )2 ( )2 ( )2

n
1 ∑
n ∑
n
< ak bk + ak bk .
2
k=1 k=1 k=1

1.37. We prove the second inequality. The first inequality can be proved
similarly. ∑n ∑n
The sum ( k=1 ak ) ( k=1 bk ) contains n2 terms of two kinds
∑n
(i) n terms of the form ak bk . Their sum is written as k=1 ak bk ;
n(n − 1) terms of the form ai bj with i ̸= j. Their sum is written as
(ii) ∑
i̸=j ai bj .

The terms in (ii) can be grouped as ai bj + aj bi . For every such pair we can
write

ai bj + aj bi = (ai bi + aj bj ) − (ai − aj )(bi − bj ) < ai bi + aj bj .


∑ ∑n
Thus the sum i̸=j ai bj is less than (n − 1) k=1 ak bk .

1.38. We prove the second inequality. Denote σ(a1 , a2 , . . . , an ) =


(a′1 , a′2 , . . . a′n ). If (a′1 , a′2 , . . . a′n ) = (a1 , a2 , . . . , an ), we have equality. Suppose
(a′1 , a′2 , . . . a′n ) ̸= (a1 , a2 , . . . , an ). Then there exists i = min{1, 2, . . . , n | ai <
a′i }, that is, the smallest i for which we have an inequality. Then there exists
j > i with a′j = ai < a′i . Define (a′′1 , a′′2 , . . . a′′n ) such that

 ′
ak , k ∈
/ {i, j}
′′ ′
ak = aj , k = i,

 ′
ai , k = j.
∑n ∑n
Denote S ′ = i=1 a′i bi and S ′′ = i=1 a′′i bi . Then

S ′ − S ′′ = bi (a′i − a′′i ) + bj (a′i − a′′i ) = bi (a′i − a′j ) + bj (a′i − a′j )


= (a′i − a′j )(bi − bj ) ≤ 0.

Thus S ′ ≤ S ′′ . We repeat this process at most n times and the second


inequality is proved.
The first inequality can be proven similarly.

1.39. (i) The inequality can be proven by induction. For n = 1 one has an
identity. For n = 2 one has


2 ∑
2
(1 − ak ) = 1 − a1 − a2 + a1 a2 ≥ 1 − a1 − a2 = 1 − ak .
k=1 k=1

Suppose the inequality is true for n = m. Then


1.1 Solutions 17


m+1 ∏
m ∏
m ∏
m
(1 − ak ) = (1 − am+1 ) (1 − ak ) = (1 − ak ) − am+1 (1 − ak )
k=1 k=1 k=1 k=1

m ∑
m+1
≥ (1 − ak ) − am+1 ≥ 1 − ak .
k=1 k=1

Thus inequality (1.23) is proved for every n.


(ii) It follows from (i).

1.40. The inequality


( )2
3 1 1
0 ≤ x − x + = (x + 1) x −
3
4 4 2

holds for x ≥ −1. Substituting x1 , x2 , . . . , xn we obtain


n (
∑ ) ∑ ∑ 3∑
n n n
3 1 3 n n
0≤ x3i − xi + = x3i − xi + =0− xi + ,
i=1
4 4 i=1 i=1
4 4 4 i=1 4
∑n
so i=1 xi ≤ n/3. Equality holds only in the case when n = 9k, k of the
numbers x1 , x2 , . . . , xn are −1, and 8k of them are 1/2.

1.41. If any ak or bk is zero, the inequality is trivial. Suppose that all entries
are positive. By the mean inequality, we have
a1 a2 an
√ + + ··· +
a 1 a 2 a n a + b1 a2 + b2 an + bn
n ... ≤ 1
a1 + b1 a2 + b2 an + bn n
√ b1 b2 bn
+ + ··· +
b1 b2 b n a + b a + b a n + bn
n
... ≤ 1 1 2 2
.
a1 + b1 a2 + b2 an + bn n
Add side by side the two inequalities and the result follows.

1.42. First approach. The identities follow by the Moivre formula from
(√ ( π π ))n ( nπ nπ )
(1 + i)n = 2 cos + i sin = 2n/2 cos + i sin .
4 4 4 4
Second approach. One proves it by induction.
One ca try using the following Mathematica commands r
TraditionalForm[Sum[(-1)^k Binomial[n, 2*k], {k, 0, n/2},
Assumptions \[RightArrow] n \[Element] Integers &&
n \[GreaterSlantEqual] 0]]
TraditionalForm[Sum[(-1)^k Binomial[n, 2*k + 1], {k, 0, n/2},
Assumptions \[RightArrow] n \[Element] Integers &&
18 1 Sets and Numbers

n \[GreaterSlantEqual] 0]

1.43. One has


{
a1 − a2 = a2 − a3 =⇒ a2 = (a1 + a3 )/2,
|a1 − a2 | = |a2 − a3 | =⇒
a1 − a2 = a3 − a2 =⇒ a1 = a3 .

Consider thepartition Sn = Sn1 ∪ Sn1 ∪ Sn3 , where


Sn1 is theset of triples so that a1 , a3 are even and distinct,
Sn2 is theset of triples so that a1 , a3 are odd and distinct,
Sn3 is theset
⌊ of triples
⌋ so that a1 = a3 ∈ A. Therefore |Sn3 | = (n + 1)2 .
n+2
There are even numbers in A. Therefore
2
⌊ ⌋ (⌊ ⌋ )
n+2 n+2
|Sn | =
1
−1 .
2 2
⌊ ⌋
n+1
There are odd numbers in A. Therefore
2
⌊ ⌋ (⌊ ⌋ )
n+1 n+1
|Sn2 | = −1 .
2 2

Hence
⌊ ⌋ (⌊ ⌋ ) ⌊ ⌋ (⌊ ⌋ )
n+2 n+2 n+1 n+1
|Sn | = (n + 1)2 + −1 + −1
2 2 2 2
⌊ n ⌋ (⌊ n ⌋ ) ⌊ n − 1 ⌋ (⌊ n − 1 ⌋ )
= (n + 1)2 + +1 + +1 .
2 2 2 2

1.44. We use the theorem that every nonempty subset of N has a smallest
element in respect to the natural ordering, i.e., N is well-ordered.
Suppose there exists a ∈ N such that f (a) ̸= g(a). Then the set

A = {n | n ∈ N, f (n) ̸= g(n)}

is nonempty. Let B be the image of A by g, that is B = {g(n) | n ∈ A}. B is


nonempty and let g(a) be its smallest element, where a ∈ A. By hypothesis,
g(a) ≤ f (a). Since a ∈ A, we have that g(a) < f (a). Since f is onto, there
exists b ∈ N such that f (b) = g(a) < f (a). Note that b ̸= a. Since g is
one-to-one,

g(b) ̸= g(a) = f (b) < f (a).


1.1 Solutions 19

From here it follows that b ∈ A and g(b) < g(a) = f (b) < f (a). Now
the minimality of g(a) is violated. So our assumption does not hold and we
conclude that f = g.

1.45. We are looking for functions satisfying the condition that there exists
a nonnegative real m so that |f (k)| ≤ m for all k ∈ Z.
Suppose there exists f a solution. For n = k = 0, we have

2f (0) = 2f 2 (0) =⇒ f (0) ∈ {0, 1}.

Thus we have two cases either f (0) = 0 or f (0) = 1.


Suppose f (0) = 0. For n = 0, we have 2f (k) = 0, for all k ∈ Z. So f is
the null function. Conversely, the null function satisfies the hypothesis. Hence
the null function is a solution to this exercise.
Suppose f (0) = 1. For k = 0, we have

f (n) + f (−n) = 2f (n), for all n ∈ Z.

So f is even. For n = k, we have f (2n) = 2(f (n))2 − 1, for all n ∈ Z.


Suppose |f (1)| ≥ 2. Then by induction one can show that

|f (2n )| ≥ 2n+1 , for all n ∈ Z.

But this contradicts the boundedness of f.


So f (1) ∈ {−1, 0, 1}.
Suppose f (1) = 1. For n = 1, we have

f (k + 1) = 2f (k) − f (k − 1), for all k ∈ Z.

From here we immediately have that f (n) = 1, for all n ∈ Z and this f is a
solution of the exercise.
Suppose f (1) = −1. Then for n = 1, we have

f (k + 1) + 2f (k) + f (k − 1) = 0, for all k ∈ Z.

Immediately we have that f (k) = (−1)k , for all k ≥ 0, and so f (k) = (−1)k ,
for all k ∈ Z. One can check that this function is a solution of the exercise.
Suppose f (1) = 0. For n = 1, we have

f (k + 1) + f (k − 1) = 0 =⇒ f (k + 1) = −f (k − 1), for all k ∈ Z.

Since f (0) = 1 and f (1) = 0, we find that

f (4k) = 1, f (4k + 1) = 0, f (4k + 2) = −1, f (4k + 3) = 0, for all k ∈ Z.

Conversely, one can check that the function f : Z → Z given by


20 1 Sets and Numbers


1, n = 4k,

0, n = 4k + 1,
f (n) =

−1, n = 4k + 2,


0, n = 4k + 3

is a solution to our exercise.

1.46. We start by proving that f (0) < f (1) < f (2) < . . . by induction. Let
Pn be the statement that f (n) is the smallest element of {f (n), f (n+1), . . . }.
For m > 0, by assumption, f (m) > f (s), where s = f (m − 1). So f (m) is
not the smallest element in {f (0), f (1), f (2), . . . }. This set being a nonempty
subset of N, has a smallest element. Hence the unique smallest element is
f (0). Thus P0 is true.
Suppose Pn is true. Take m > n + 1. Then m − 1 > n and so f (m − 1) >
f (n). We also have that

f (0) < f (1) < f (2) < · · · < f (n).

So, f (n) ≥ n + f (0) ≥ n. Hence f (m − 1) ≥ n + 1. So f (m − 1) belongs


to {n + 1, n + 2, . . . }. By hypothesis, f (m) > f (f (m − 1)), so f (m) is not
the smallest element of {f (n + 1), f (n + 2), . . . }. This set being a nonempty
subset of N, has a smallest element. So f (n + 1) must be the unique smallest
element. Thus we showed that Pn+1 is true. So, Pn is true for all n. So we
have

0 ≤ f (0) < f (1) < f (2) < f (3) < . . . . (1.4)

It also follows that f is increasing, i.e. if n ≤ m, then f (n) ≤ f (m).


Suppose that for some m ∈ N, we have f (m) > m. Then

f (m) ≥ m + 1 =⇒ f (f (m)) ≥ f (m + 1).

This contradicts the assumption. Hence

f (n) ≤ n, for all n ∈ N. (1.5)

From (1.4) and (1.5) we conclude that if f satisfies the assumptions, then

f (n) = n, for all n ∈ N.

We can check easily that this function satisfies all assumptions. So, the
only one solution to our exercise is f (n) = n, for all n ∈ N.

1.47. Under the form in the statement, the exercise has no solution since
in the right hand side occurs 00 , which is rejected. Instead some cases are
1.1 Solutions 21

interesting, the method that we use here in the main case is based on the
separation of variables.
(a) Consider the simplest case

f (x) · f (y) = f (x/2) + f (y/2) , for all x, y ∈ [0, ∞[ . (1.6)

Substitute x = y = 0 in (1.6) to get either f (0) = 0 or f (0) = 2.


Substitute y = x in (1.6) to get

f 2 (x) = 2 · f (x/2) or f (x/2) = 2−1 f 2 (x).

Now substitute the last result in (1.6) to get

(f (x) − f (y))2 = 0 =⇒ f (x) = f (y), for all x, y ∈ [0, ∞ [ .

So f is a constant function, either f (x) = 0, for all x ∈ [0, ∞[ , or f (x) = 2,


for all x ∈ [0, ∞[ . We check both constant functions are solutions to (1.6).
(b) Suppose that (1.24) has the form

f (x) · f (y) = y α f (x/2) + f (y/2) , for all x, y ∈ [0, ∞ [ , (1.7)

where α ̸= 0.
For y = 0, we have

f (x) · f (0) = f (0) , for all x ∈ [0, ∞[ .

Thus either f (0) = 0 or f (x) = 1, for all x ∈ [0, ∞ [ . We reject the case
f (x) = 1, for all x ∈ [0, ∞[ , it does not satisfy (1.7). For x = 0, we have

f (y) · f (0) = y α f (0) + f (y/2) , for all y ∈ [0, ∞[


=⇒ f (y/2) = f (0), for all y ∈ [0, ∞ [ .

So we get the null function which satisfies (1.7).


(c) Suppose α ̸= 0 ̸= β. Suppose such a function f does exist. We have

x = y = 0 =⇒ f (0) · f (0) = 0 =⇒ f (0) = 0.

The left hand side (1.24) is symmetric in x and y, so the right hand side has
to be as well. Therefore

y α f (x/2) + xβ f (y/2) = xα f (y/2) + y β f (x/2)


=⇒ (xα − xβ )f (y/2) = (y α − y β )f (x/2) , for all x, y ∈ [0, ∞[ .

If α ̸= β, then for x, y > 0, and x, y ̸= 1, one has

f (x/2) f (y/2)
= α .
x −x
α β y − yβ
22 1 Sets and Numbers

So the function
f (x/2)
x 7−→
xα − xβ
is constant on ]0, 1[ ∪ ]1, ∞ [ . This implies that there is a real constant c such
that

f (x) = c(2α xα − 2β xβ ), for all x ∈ ]0, 1/2[ ∪ ]1/2, ∞[ .

Under these conditions the functional equation (1.24) has the form

c2 (2α xα − 2β xβ )(2α y α − 2β y β ) = cyα(xα − xβ ) + cxβ(y α − y β )


= c(xα y α − xβ y β ).

Fix y and consider the last relation only on x. Then

2α c2 (2α y α − 2β y β )xα − 2β c2 (2α y α − 2β y β )xβ = cy α xα − y β xβ .

Equating the coefficients of xα and xβ we have


{
2α c2 (2α y α − 2β y β ) = cy α ,
for all y ∈ ]0, 1/2[ ∪ ]1/2, 1[ ∪ ]1, ∞[ .
2β c2 (2α y α − 2β y β ) = cy β ,

It follows that c = 0. So

f (x) = 0, for all x ∈ ]0, 1/2[ ∪ ]1/2, 1[ ∪ ]1, ∞[ .

Taking x = y = 1 and then x = y = 2 in the initial equation (1.24), we have


{
(f (1))2 = 2f (1/2) ,
(f (2))2 = (2α + 2β )f (1) = 0 =⇒ f (1) = f (1/2) = 0.

Thus we have that f (x) = 0, for all x ≥ 0.


Conversely, the null function satisfies the equation (1.24).
Suppose now that α = β. Then (1.24) becomes

f (x) · f (y) = y α f (x/2) + xα f (y/2) , for all x, y ∈ [0, ∞[ . (1.8)

For x = y = 0, we get f (0) = 0. For x = y we have

(f (x))2 = 2 · xα f (x/2) =⇒ f (x/2) = (1/2)2x−α (f (x))2 .

Then for all x, y > 0, we have


1 α −α 1
f (x) · f (y) = y x (f (x))2 + xα y −α (f (y))2
2 2
1.1 Solutions 23
α α
=⇒ (y/x) (f (x))2 − 2f (x)f (y) + (x/y) (f (y))2 = 0
2
=⇒ (y α f (x) − xα f (y)) = 0
=⇒ exists c ∈ R such that for all x > 0, f (x) = cxα .

Substitute it in (1.8) to get


α
c2 xα y α = 2c (xy/2) , for all x, y > 0.

Then either c = 0 or c = 21−α .


Conversely, the null function and f (x) = 2(x/2)α are solutions of (1.8).
We bring together the results
{
f (x) = 0,
α = β = 0 =⇒ for all x ≥ 0;
f (x) = 2
{
f (x) = 0,
α = β ̸= 0 =⇒ α for all x ≥ 0;
f (x) = 2 (x/2)
α = 0, β ̸= 0 =⇒ f (x) = 0, for all x ≥ 0;
α ̸= 0, β = 0 =⇒ f (x) = 0, for all x ≥ 0;
α ̸= 0, β ̸= 0 =⇒ f (x) = 0, for all x ≥ 0.

1.48. Suppose f is a solution. Then for y = 0 we get that for any x ∈ R,


f (x − f (0)) = 1 − x. Denote t = x − f (0). Then x = t + f (0) and f (t) =
1 − t − f (0) = 1 − f (0) − t. So, for all real x we have f (x) = −x + k, where
k is a constant. Substitute f (x) = −x + k in (1.25) to get k = 1/2. Hence
f (x) = −x + 1/2 is the only one solution of (1.25).

1.49.
Consider A = ]0, 1]. Show that int A = ] 0, 1[ , cl A = [0, 1], and A′ =
[0, 1].

1.50. Consider A = ]0, 1] ∩ Q. Show that inf A = 0, sup A = 1, int A = ∅,


cl A = [0, 1], A′ = [0, 1], and fr A = [0, 1].

1.51. We approach the inequalities geometrically. Consider Figure 1.2. Since


the sinus and the tangent functions are odd, it is just enough to prove

0 < sin x < x < tan x, 0 < x < π/2. (1.9)

Consider a circle centered in O of unitary radius, i.e. OC = OA = 1. Let x be


the measure of the angle AOC[ (x measured in radians). Then the length of
the arc AC is equal to x and AC = sin x while AB = tan x. Since triangle
OAC is contained in sector OAC which, in turn, is contained in triangle
24 1 Sets and Numbers

Bbc
C
bc
x

x
bc bc
A
O

Fig. 1.2. Figure for Exercise 1.51

OAB, their areas satisfy


sin x x tan x
0< < < .
2 2 2
From here the inequalities in (1.9) follow.
2
Vector Spaces and Metric Spaces

2.1. R is a vector space over Q because each rational number is a real number
and R is a commutative field. Q is not a vector space over R because the
product between a scalar (in Q ) and a vector (in R) generally does not belong
to Q.

2.2. The sum of two polynomials with real coefficients is a polynomial with
real coefficients. The product of a real number by a polynomial with real coef-
ficients is a polynomial with real coefficients. The null element is the constant
null polynomial.

2.3. The first answer follows from the previous exercise. Since {1, x, . . . , xn }
is a basis, the dimension of this vector space is n + 1.

2.4. No. For example, the sum of the first degree polynomials p(x) = x by
q(x) = −x does not belong to the set of first degree polynomials.

2.5. Suppose x, y ∈ c0 and a, b ∈ R. Then x = (x1 , x2 , . . . ) and y =


(y1 , y2 . . . . ) with xn , yn → 0 as n → ∞. We have
n→∞
|axn + byn | ≤ |a| · |xn | + |b| · |yn | ≤ max{|a|, |b|}(|xn | + |yn |) −−−−→ 0.

Hence ax + by ∈ c0 .

2.6. For x = (x1 , x2 , . . . ) ∈ c0 , because of |xk | ≥ 0 for all k ∈ N, it is clear


that ∥x∥ ≥ 0. If ∥x∥ = 0,

|xk | = 0, for all k ∈ N, =⇒ xk = 0, for all k ∈ N, =⇒ x = 0.

The positive homogeneity of the norm follows from


26 2 Vector Spaces and Metric Spaces

∥αx∥ = sup |αx| = |α| · ∥x∥.


k

The triangle inequality of the norm follows from

∥x + y∥ = sup |xk + yk | ≤ sup(|xk | + |yk |) ≤ sup |xk | + sup |yk |


k k k k
≤ ∥x∥ + ∥y∥.

2.7. Suppose that the claim is not true. Then for every positive integer k
there exists a system {x1k , x2k , . . . , xnk } of linear dependent vectors ful-
filling ∥xi − xik ∥ < 1/k, i = 1, 2, . . . , n. Then there exist some coefficients
c1k , c2k , . . . , cnk so that

c1k x1k + c2k x2k + · · · + cnk xnk = 0.


∑n 2
We may suppose i=1 cik = 1 for all k. Since the boundary of the closed
unit ball in Rn is compact, the system of vectors (c1k , c2k , . . .∑ , cnk )k has a
n 2
convergent subsequence tending to a vector (c1 , c∑ 2 , . . . , cn ) with k=1 ck = 1.
n
Obviously, lim xik = xi , i = 1, 2, . . . , n. Thus i=1 ci xi = 0, contradicting
the hypothesis.

2.8. Let rn be the radius of Bn . By the assumption, for m > n we have


∥xn − xm ∥ + rm ≤ rn , or ∥xn − xm ∥ ≤ rn − rm . Immediately follows that
rm ≤ rn , for m > n, that is the sequence of radii is non-increasing. So it
has a lower bound. Denote r = inf rn . If r > 0, there exists a rank k with
xk = xn for all n ≥ k and thus exists lim xn . If r = 0, the sequence (xn ) is
fundamental and again exists lim xn . The proof is the same if one considers
closed balls.

2.9. int A = ]0, 1[ , cl A = [0, 1] ∪ {2}, A′ = [0, 1], and fr A = {0, 1, 2}.

2.10. The set A is neither open nor closed.


3
Sequences and Series

3.1. Consider the sequence (an ) of all natural numbers, but not perfect
squares, i.e., a1 = 2, 2 = 3, a3 = 5, a4 = 6, . . . . Set
m
xk,m = (ak )2 , for k = 1, 2, . . . , and m = 0, 1, 2, . . . .

Then

xk,m+1 = x2k,m

and to each n > 1 corresponds a pair k, m such that n = xk,m . Define


{
xk+1,m , k odd
f (0) = 0, f (1) = 1, f (xk,m ) =
xk−1,m+1 , k even,

for k = 1, 2, . . . , and m = 0, 1, 2, . . . . Then

f (f (n)) = n2 , for all n ∈ N∗ .

3.2. Indeed, it is enough to remark that the sequence has positive terms, so
it is monotonically increasing, but divergent. To check this the last property
it is enough to see that it is not a Cauchy one. The sequence (an ) is not
fundamental because

a2n − an = 1/(n + 1) + · · · + 1/(2n) > 1/(2n) + · · · + 1/(2n) = 1/2.

3.3. Suppose (an ) is bounded. Then there exists a positive M such that
|an | ≤ M, for every n. From the hypothesis we infer that the open intervals
]an − 1/(2n), an + 1/(2n)[ are disjoint and their union satisfies
28 3 Sequences and Series

∪n ]an − 1/(2n), an + 1/(2n) [ ⊂ ] − M − 1/2, M + 1/2 [ .

Since the length of the interval ] − M − 1/2, M + 1/2 [ is 2M + 1, it follows


that the sum of the lengthes to the disjoint intervals ]an −1/(2n), an +1/(2n)[
does not exceed 2M + 1. On the other side, the length of an interval
]an − 1/(2n), an + 1/(2n)[∑n is equal to 1/n. So the sum of the lengthes of
the first n intervals is k=1 1/k which tends to +∞, accordingly to (a).
The contradiction shows that our hypothesis regarding the boundedness of
the sequence (an ) is false. Hence the sequence is unbounded.

3.4. We note that xn > 0. Moreover, xn+1 − a ≥ 0, (and thus (ii) is proved)
and xn+1 − xn ≤ 0. Then (xn ) is monotonic and bounded,
√ hence convergent.
Passing to the limit in (3.82), we find that xn → a as n → ∞.

3.5. If x = 0, the result is clear. Suppose x ̸= 0 and denote an = |x|n . Then


0 < an+1 < an . Therefore (an ) converges to a number, say l. Passing to the
limit in recursion an+1 = an |x|, we find that l = l · |x|. The only solution is
l = 0, the limit of (an ). But lim |x|n = 0 implies lim xn = 0.

3.6. If α ≤ 0, the conclusion follows from the previous exercise. Suppose


α > 0. Obviously ( )α
1 1
xn+1 = 1 + xn .
n p+1
We can find a constant c ∈ ] 1/(1 + p), 1[ so that xn+1 ≤ cxn , for large n.
Then the conclusion follows.

3.7. We remark that (xn ) has positive terms and is increasing since for all
n ≥ 1,

√ √
x2 = a + a > a = x1 , (xn+2 − xn+1 )(xn+2 + xn+1 ) = xn+1 − xn .

We are interested√now to see if the sequence √is bounded above or not.


Since√x1 < (1 + 1 + 4a)/2 and if xn < (1 + 1 + 4a)/2, then xn+1 <
(1 + 1 + 4a)/2, for all n ≥ 1, we conclude that the sequence is bounded.
Hence the given sequence is convergent.
√ √
3.8. First√approach. We remark that 1 ≤ 2 < 2, 1 ≤ 2 < a2 < 2,
. . . , 1 ≤ 2 < an < 2, . . . , and that (an ) is increasing. Then there exists
l = lim an . Passing to the limit in the recurrence relation, one gets l = 2.
Second approach. We invoke the identity
√ √
√ π
2 + 2 + · · · + 2 = 2 cos n+1
2
and pass to the limit in both sides.
3.1 Solutions 29

3.9. We introduce other two sequences (bn ) and (yn ) by


n
bn = an /e2 , n ≥ 1,
√ √ √
√ √ √
y1 = b1 , y2 = b 1 + b 2 , . . . , yn = b1 + b2 + · · · + bn , . . . ,

and we note that for every n ∈ N∗ yn = xn /e, i. e., (xn ) and (yn ) converge
or diverge simultaneously. We also remark that (yn ) is increasing.
From the hypothesis it follows that there exists an n0 ∈ N∗ such that for
n ≥ n0 ,
n
(1/n) ln(ln an ) < ln 2 ⇐⇒ an < e2 ⇐⇒ bn < 1.
Consider a = max{b1 , b2 , . . . , bn0 , 1} and define the following sequence

√ √ √
z1 = a, z2 = a + a , . . . , zn+1 = a + zn , . . . .

Then yn ≤ zn . Based on the Exercise (3.7), we deduce that (zn ) converges.


Therefore (yn ) converges and, at the end, (xn ) converges, as well.

3.10. If a = b, an = bn = a, for every n. Thus all the conclusions follow at


once. Suppose a ̸= b. By induction one can show

0 ≤ bn < bn+1 < an+1 < an , 0 < an+1 − bn+1 < (an − bn )/2, n ≥ 1,

and the conclusions follow.

3.11. All the terms are positive. Suppose there exists n ∈ N∗ such that
an > bn . Then
an+1 2bn
= < 1 =⇒ an+1 < an ;
an an + bn
( )2
b2n+1 2an bn bn+1 2an
= =⇒ = > 1 =⇒ bn+1 > bn ;
bn an + bn bn an + bn

bn+1 an+1
= > 1 =⇒ an+1 > bn ;
bn bn

an+1 an+1 an+1
=√ = > 1 =⇒ an+1 > bn+1 .
bn+1 an+1 bn bn

From the above inequalities we conclude that (an ) decreases, (bn ) increases,
and an < bn , for all n ∈ N∗ . Hence the two sequences converge. Passing to
the limit in the recursion of bn , we get that the two limits coincide. Why the
common limit is equal to 2π is shown at page 356.

3.12. The first limit is equal to 0 since


30 3 Sequences and Series

n/(n2 + n) ≤ 1/(n2 + 1) + 1/(n2 + 2) + · · · + 1/(n2 + n) ≤ n/(n2 + 1).

One can try using one of the Mathematica r commands


Limit[Sum[1/(n^2 + i), {i, 1, n}], n -> Infinity]
Limit[\!\(\*SubsuperscriptBox[\(\[Sum]\),
\(i = 1\), \(n\)]\(1/\((n^2 + i)\)\)\), n -> Infinity]
For the second limit we take into account the following inequalities

1 + 2 2 + · · · + nn 1 + n + n2 + · · · + nn nn+1 − 1 1 n→∞
1≤ ≤ = −−−−→ 1.
n n n n n − 1 nn
Thus we get that the limit is 1.
For this limit we can also use Theorem 1.22 of Stolz–Cesaro.

3.13. (a) We note that xn > 0 for all n ∈ N∗ \ {1} and xn < 1 for all n ∈ N∗ .
We can improve the upper of the sequence since
√ √
−1 + 5 −1 + 5
xn < =⇒ xn+1 < , for all n ∈ N∗ .
2 2
Using the previous result, it is clear that xn+1 > xn . Thus the sequence
is monotone and bounded, therefore√convergent. Passing to the limit in the
recurrence, we find the limit (−1 + 5)/2.
(b) It converges to 5. √
(c) It converges to (−1 + √5)/2.
(d) It converges to (−1 + 5)/2.
(e) If p = 0, it results the constant sequence xn = 1, for all. So the sequence
converges to 1. If p = 1, the general term of the sequence is equal to xn+1 =
n+a, thus the sequence diverges. If p ∈ ]0, 1 [ , the general term of the sequence
is equal to xn+1 = 1 + p + · · · + pn−1 + apn , thus the sequence converges to
1/(1 − p).

3.14. We note that the two sequences are unbounded. Then there exist n1 , n2
satisfying |an1 − an2 | > 2 (since otherwise (an ) is bounded). Further, there
exists n3 such that |bn1 − bn3 | > 1 and |bn2 − bn3 | > 1 (since otherwise the
sequence (bn ) is bounded). Now, if |an1 −an3 | > 1, then n = n1 and m = n3 .
If |an1 − an3 | ≤ 1, then |an2 − an3 | > |an1 − an2 | − |an1 − an3 | > 1, and hence
n = n2 and m = n3 .

3.15. Note that always lim inf n→∞ an ≤ lim inf n→∞ ak n for any subse-
quence (ak n ). We have lim inf n→∞ (an + bn ) = limn→∞ (ap n + bp n ) and
lim inf n→∞ ap n = limn→∞ am p n . By the previous note

lim inf an + lim inf bn ≤ lim inf ap n + lim inf bp n


n→∞ n→∞ n→∞ n→∞
= lim am p n + lim inf bp n ≤ lim am p n + lim inf bm p n .
n→∞ n→∞ n→∞ n→∞
3.1 Solutions 31

Since the sequence (am p n +bp n )n is a subsequence of the convergent sequence


(ap n + bp n )n , we have

lim (ap n + bp n ) = lim (am p n + bp n ).


n→∞ n→∞

Since sequence (am p n )n is convergent, the sequence (bm p n )n is also conver-


gent and thus
lim inf bm p n = lim bm p n .
n→∞ n→∞

Now we write

lim inf an + lim inf bn ≤ lim am p n + lim bm p n = lim (am p n + bm p n )


n→∞ n→∞ n→∞ n→∞ n→∞
= lim (ap n + bp n ) = lim inf (an + bn ).
n→∞ n→∞

Thus the first inequality in (3.83) holds.


From
lim inf (−bn ) = − lim sup bn ,
n→∞ n→∞

we have

lim inf (an + bn ) − lim sup bn = lim inf (an + bn ) + lim inf (−bn )
n→∞ n→∞ n→∞ n→∞

≤ lim inf ((an + bn ) + (−bn )) = lim inf an .


n→∞ n→∞

Thus the second inequality in (3.83) holds. Similarly can be proved (3.84).

3.16. (a) Take an = pn and bn = n. The sequence (bn ) is strictly mono-


tone and unbounded. So, there are fulfilled the assumptions of Stolz–Cesaro
Theorem. Because
n→∞
(an+1 − an )/(bn+1 − bn ) = pn+1 − pn = pn (p − 1) −−−−→ ∞,
n→∞
we conclude that pn /n −−−−→ ∞.
One can try using the Mathematica r command
Limit[p^n/n, n -> Infinity, Assumptions -> 1 < p]
(b) Applying Theorem 1.22 of Stolz–Cesaro, we get

(n + 1)p np + pnp−1 + . . . n→∞ 1


= −−−−→ .
(n + 1) p+1 − (n) p+1 p
(p + 1)n + . . . p+1

One can try using one of the Mathematica r commands


Limit[Sum[i^p, {i, 1, n}]/n^{p + 1}, n -> Infinity,
Assumptions -> 0 <= p && p \[Element] Integers]
(c) One can try using the Mathematica r command
32 3 Sequences and Series

Limit[Log[n]/n^k, n -> Infinity,


Assumptions -> 1 <= k && k \[Element] Integers]
(g) First we note that if a = b = 0, lim cn = 0. Indeed, there is M > 0 so
that |bn | < M. Further, lim |an | = 0. Then, by Corollary 1.5, one has

lim |cn | ≤ M lim(|a1 | + · · · + |an |)/n = M lim |an | = 0.

Suppose that a = 0, while b ̸= 0. Since bn → b, |bn | → |b|. By Corollary 1.5

lim(|b1 | + |b2 | + · · · + |bn |)n = |b|.

Denote
ηn = (|b1 | + |b2 | + · · · + |bn |)/n − |b|
and choose a positive ε. Then there is a natural N so that for n > N,
|an | < ε/(4b). Then for n > N,

|cn | < (|a1 bn + · · · + aN bn−N +1 |)/n + ε(|b| + |ηn |)/(4b).

There is a bound M > 0 for {|bn | | n ∈ N∗ }. For a positive ν ∈ N∗ we have

|ηn | < |b| and M (|a1 | + · · · + |aN |)/n < ε/2, for all n > ν.

Then |cn | < ε and this case is proved.


Suppose ab ̸= 0. Consider αn = an − a. Then

cn = a(b1 + · · · + bn )/n + dn (3.1)

where
dn = (α1 bn + α2 bn−1 + · · · + αn b1 )/n.
Since αn → 0, we have that dn → 0, too. We apply Corollary 1.5 to (3.1)
and we find that cn → ab, as n → ∞.

3.17. We denote by M the set of constants α for which the above limit exists
and it is finite. Remark that M ̸= ∅ since by Proposition 1.18, 1 ∈ M. Now


γ, α = 1,
1 1 n→∞
1 + + · · · + − ln n + (1 − α) ln n −−−−→ +∞, α < 1,
2 n 

−∞, α > 1.

Hence M = {1}.

3.18. Since the set {a1 , a2 , . . . , an } is finite, max{a1 , a2 , . . . , an } is attained


and is a member of the set. Denote ak = max{a1 , . . . , an }. If a1 = · · · = an ,
then max{a1 , a2 , . . . , an } = a1 .
3.1 Solutions 33

Suppose not all the members are equal. Suppose k ∈ {2, . . . , n − 1} and
a1 < ak , ak > an . Then

a2k ≤ ak−1 ak+1 ≤ max am · max am = a2k


m=1,...,n m=1,...,n

and thus
a2k = ak−1 ak+1 .
One has ak ≥ ak−1 and ak ≥ ak+1 . Thus we get ak−1 = ak = ak+1 . Using
the same argumentation, we find that all the elements of the set are equal,
which is a contradiction. Thus the maximum is equal to a1 or an . Both results
are possible. Suppose a1 = 1, a2 = 2, a3 = 3. Then max{a1 , a2 , a3 } = a3 .
Suppose now a1 = 3, a2 = 2, a3 = 1. Then max{a1 , a2 , a3 } = a1 .

3.19. From xn+1 = xn (xn −1)+1 we have that xn > 1, for all n ≥ 1 and from
xn+1 −xn = (xn −1)2 > 0 we conclude that the sequence is increasing. Suppose
it is bounded. Then it converges and let limn→∞ xn = α > 1. From the
recurrence we find out that α = 1, a contradiction. Thus (xn ) monotonically
diverges to ∞. Since xn+1 − 1 = xn (xn − 1), then
1 1 1 1 1 1 1
= = − =⇒ = − .
xn+1 − 1 xn (xn − 1) xn − 1 xn xn xn − 1 xn+1 − 1

Thus the general term of the sequence of partial sums can be written as
1 1 1
sn = + + ··· +
x1 x2 xn
1 1 1 1 1 1
= − + − + ··· + −
x1 − 1 x2 − 1 x2 − 1 x3 − 1 xn − 1 xn+1 − 1
1 1
= − .
x1 − 1 xn+1 − 1
n→∞
Since (xn ) diverges to ∞, we conclude sn −−−−→ 1/(a − 1) and therefore



1/xn = 1/a − 1.
n=1

3.20. The sequence (xn ) is convergent. Let x = lim xn . The general term
of the series is nonnegative. Thus the series is convergent if and only if the
sequence of partial sums is bounded. The general terms of the sequence of
partial sums can be written as

sn = (1 − x1 /x2 ) + (1 − x2 /x3 ) + · · · + (1 − xn /xn+1 )


34 3 Sequences and Series
v
u n ( )

n
u∏ √
≤n− t
n
=n− xk /xk+1 n (xk /xk+1 ) = n 1 − x1 /xn+1
k=1 k=1

e(1/n) · ln(x1 /xn+1 ) − 1 x1 n→∞ x


=− · ln −−−−→ ln .
(1/n) · ln(x1 /xn+1 ) xn+1 x1

3.21. (i) It diverges because is the generalized harmonic series with p = 1/2.
(ii) It converges. One can check it by the root of ratio tests.
(iii) It diverges.
(iv) It diverges because is the generalized harmonic series with p = 1/3.
(v) It converges based on the comparison, ratio, or root test. Also try the
Mathematica r command
Sum[(n + 1)/( n^2 3^n), {n, 1, Infinity}]
(vi) It converges to 5 sin(x)/(26−10 cos x). Try the Mathematica r command
Sum[Sin[n * x]/5^n, {n, 1, Infinity}]
(vii) It converges to 1. Try the Mathematica r command
Sum[2/3^n, {n, 1, Infinity}]

(viii) The series converges.


(ix) It converges to 1 because the partial sum is 1 − 1/(n + 1). Also try the
Mathematica r command
Sum[1/(n (n + 1)), {n, 1, Infinity}]
(x) It converges to 1. Also try the Mathematica r command
Sum[((-1)^(n - 1)/(n + 1)) (2 + 1/n), {n, 1, Infinity}]
(xi) It converges. Try the Mathematica r command
Sum[ArcTan[1/(2 n^2)], {n, 1, Infinity}]

3
(xii) Write the partial sum of the series to conclude that it converges to 1− 2.
r
(xiii) It converges to ln 2. Try the Mathematica command
Sum[Log[(n^2 + 5 n + 6)/(n^2 + 5 n + 4)], {n, 1, Infinity}]
∑∞ −√n
(xiv) The series n=1 e√n has positive terms and since e−x < x−3/2 for all
∑ −2
positive x, is majorized by the generalized harmonic series n .
(xv) The series diverges for p ≤ 0, converges for p ∈ ]0, 1], and converges
absolutely for p > 1.
(xvi) The series converges by the root test.
(xvii) The series diverges.

3.22. (a) Its domain of convergence is the set ]1, ∞[ , according to Theorem
3.10. On this set the series converges even absolutely. Try the Mathematica r
commands
3.1 Solutions 35

SumConvergence[1/n^x, n]

$Assumptions = {x \[Element] Reals};


SumConvergence[1/n^x, n]

$Assumptions = {x > 1};


SumConvergence[1/n^x, n]
(b) For x > 1, the series converges absolutely by (a). For 0 < x ≤ 1, it
converges by Theorem 3.22 of Leibniz. For x ≤ 0 the necessary condition
(Theorem 3.2) is not satisfied, hence the series diverges. We conclude that the
domain of convergence is ]0, ∞[ . Try the Mathematica r commands
$Assumptions = {x > 0};
SumConvergence[(-1)^n/n^x, n]
(c) Since it is periodic, we restrict our discussion to ]−π/2, π/2[ . By Theorem
3.8, the series converges absolutely on ] − π/4, π/4[ . Otherwise it diverges.
Try also the Mathematica r command
SumConvergence[(Tan[x])^n, n]
(d) For x = 0, it converges obviously. By the root test we have
/
|xn+1 | n
lim sup = |x|.
n→∞ n + 1 |x n|

So, for |x| < 1, the series converges absolutely. For x = −1, the series con-
verges by Leibniz Theorem. For x = 1, the series is the harmonic series, thus it
diverges. For |x| > 1, the necessary condition is not satisfied, thus it diverges.
(e) We have

n |x − 3|
lim sup |(x − 3)n /nn | = lim sup = 0.
n→∞ n→∞ n

Thus the given series converges for all x ∈ R. Try also the Mathematica r
command
SumConvergence[(x - 3)^n/n^n, n]
(f) By the ratio test we have


|x|, |x| < 1, x ̸= 0
xn+1 (1 + x2n ) 1 + x2n 
lim sup = |x| · lim = 1, |x| = 1
(1 + x2n+2 )xn n→∞ 1 + x2n+2 
 1
n→∞
 , |x| > 1.
|x|

For x = 0, the series converges. It also converges absolutely


∑ for all x ̸= ±1.
For x = ±1, the series diverges since it looks like (−1)n /2. Try also the
Mathematica r command
SumConvergence[x^n/(1 + x^(2 n)), n]
36 3 Sequences and Series

(g) For x = 0, the series diverges. The ration test gives no information re-
garding the convergence. By the Raabe–Duhamel test, i.e., Theorem 3.14, we
have
( )
n! (x2 + 1)(x2 + 2) · · · (x2 + n + 1)
lim inf n · − 1
n→∞ (x2 + 1)(x2 + 2) · · · (x2 + n) (n + 1)!
= x2 .

Thus the series converges for x2 > 1, that is x < ∑ −1 or x > 1. It diverges
for −1 < x < 1. If x2 = 1, the series looks like 1/(n + 1), so diverges.
Hence the series converges if and only if x2 > 1. Try also the Mathematica r
commands
$Assumptions = {x \[Element] Reals};
SumConvergence[n!/Product[x^2 + i, {i, 1, n}], n]

3.23. (i) x ∈ [−1, 1[ .


(ii) x ∈ ] − 1, 1].
(iii) x ∈ ] − 5, 5[ .
(iv) x = {0}.
(v) x ∈ ] − 3, √3[ . √
(vi) x ∈ [−5/ 3, 5/ 3].
(vii) x ∈ [−1, 3[ . √ √
(viii) x ∈ ] − 2 − 5, −2 + 5 [ .
(ix) x ∈ ] − 1, 5[ .
(x) x ∈ ] − 1/ max{a, b}, 1/ max{a, b} [ .

3.24. For the first series we apply the root test. Then limn→∞ n | sinn x|/nα =
| sin x|. Thus if | sin x| < 1, i.e., for all x ∈ R \ {2kπ ± π/2 | k ∈ Z}, the series
converges ∑ absolutely. Suppose x is of the form 2kπ + π/2. Then the series
becomes 1/nα , which converges
∑ if and only if α > 1. For of the form
2kπ − π/2, the series is (−1)n /nα , which converges for α > 0. Hence
if α > 1, the domain of convergence is R; if 0 < α ≤ 1, the domain of
convergence is R \ {2kπ + π/2 | k ∈ Z}; if α ≤ 0, the domain of convergence
is R \ {2kπ ± π/2 | k ∈ Z}.
For the second series we apply the ratio test. Then we get as limit of the ratio
in Theorem 3.18 the fraction |1 − x2 |/(1 + x 2
∑). For x ̸= 0, the series converges
∞ n
absolutely. For x = 0, we get the series n=2 (−1) / ln n, which converges
by Theorem 3.22 of Leibniz. Hence the domain of convergence is R. Try as
well the Mathematica r command
SumConvergence[(-1)^n/Log[n] ((1 - x^2)/(1 + x^2))^n, n]
For the third series we apply Theorem 3.22 of Leibniz for an arbitrary real x.
Because
√ √
x2 + 1/n2 ≤ |x| + 1/n and 0 ≤ x2 + 1/n2 − |x| ≤ 1/n,
3.1 Solutions 37

we conclude that the series converges on R.

3.25. The sequence (an (x)), where an (x) = 1/(x2 + n), has positive terms
and converges monotonically to 0. Since |an (x) − 0| ≤ 1/n → 0 as n → ∞,
we conclude that the sequence (an (x)) converges uniformly to 0 on R. By
Theorem
∑∞ 4.3 we have that the series converges uniformly on R. The series
2
n=1 1/(x + n) diverges for each real x, since comparing with the harmonic
series, the comparison test supplies the answer.
4
Limits and Continuity

4.1. Consider p > 0 since otherwise the necessary condition for convergence
is not satisfied. Denote f (x) = x−p , x > 0. We have that ex · f (ex )/f (x) =
xp /e(p−1)x . Thus for p > 1, limx→∞ ex · f (ex )/f (x) = 0 < 1, and we conclude
the series converges. For 0 < p < 1, ex · f (ex )/f (x) ≥ 1, and we conclude the
series diverges. The case p = 1 has been already discussed.

4.2. The function is continuous at each point in R2 \{(0, 0)} as composition of


continuous functions. Consider an arbitrary small neighborhood of the origin.
Then for x = ρ cos θ and y = ρ sin θ we have
{
ρ(cos3 θ + sin3 θ), ρ ̸= 0
f (x, y) = f (ρ cos θ, ρ sin θ) =
0, ρ = 0.

Further |ρ(cos3 θ + sin3 θ)| ≤ 2ρ. Hence the function is continuous on the real
plane.

4.3. Note the null function and the identity function are solutions to (4.54).
Thus the set of solutions is nonempty. Note also that a solution necessarily is
an odd function. We are looking for the whole set of solutions.
Taking x = y = 0, we find φ(0) = 2φ(0), i.e., φ(0) = 0. By induction
one can show that φ(n) = nφ(1), n ∈ N∗ . Then from φ(1) = φ (m/m) =
mφ (1/m) it follows φ (1/m) = 1/mφ(1) for m ∈ N∗ .
By induction one can further show that φ(p/q) = (p/q)φ(1), p ∈ Z,
q ∈ Z \ {0}. Now we know the representation of the solution if we restrict the
exercise to the system of rational numbers.
Consider an arbitrary but fixed x ∈ R \ Q. Then there exists a sequence
(qn ) of rational numbers such that qn → x. By the continuity hypothesis one
can conclude that
40 4 Limits and Continuity

φ(x) = lim φ(qn ) = lim qn φ(1) = (lim qn )φ(1) = xφ(1).

Denote a = φ(1). Then a solution of the Cauchy functional equation (4.54)


is any function φ(x) = ax, for every x ∈ R.

4.4. The constant function φ(x) = −a, x ∈ R, is a solution. Set f (t) =


φ(t) + a. Then we get the Cauchy functional equation for function f. Thus
we conclude that φ(x) = cx − a, x ∈ R.

4.5. Remark that φ(x) = ln x, x > 0, is a solution to (4.56). Set u = ex ,


v = ey , f (t) = φ(et ), and substitute them in (4.56). Then we have that f is
continuous and satisfies f (u + v) = f (x) + f (v), for u, v ∈ R. From Exercise
(4.3) we get that f (u) = au, u ∈ R. Hence φ(x) = loga x, x > 0.

4.6. Note that function φ(x) = exp(x), x ∈ R, is a solution to (4.57). Take


y = 0 in (4.57). It results that φ(x) = φ(x)φ(0) for x ∈ R. If φ(0) = 0, φ is
the zero function, contrary to our assumption. So φ(0) ̸= 0, more precisely,
φ(0) = 1.
Take y = x in (4.57). Then φ(2x) = φ2 (x) ≥ 0, for all x ∈ R. So, φ is
a non-negative function. Suppose that there is an x0 such that φ(x0 ) = 0.
Then for any real x we have

φ(x) = φ(x − x0 + x0 ) = φ(x − x0 )φ(x0 ) = 0,

contradicting that φ is a non-zero function. Thus we get that φ is a positive


function. We have the following equalities

φ(n) = φ(1 + 1 + · · · + 1) = φn (1), n ∈ N∗ ;


1 = φ(0) = φ(1)φ(−1);
φ(−n) = φn (−1), n ∈ N∗ ;
n ∈ N∗ ;
1/n
φ(1) = φ (1/n + · · · + 1/n) =⇒ φ (1/n) = (φ(1)) ,
±m/n
φ (±m/n) = (φ(1)) , m, n ∈ N∗ ;
r
φ(r) = (φ(1)) , r ∈ Q.

From the last equality taking into account the continuity assumption we infer
x
that φ(x) = (φ(1)) , for any x ∈ R. If we denote φ(1) = a(> 0), we get that
a solution to our exercise is a function of the form φ(x) = ax , x ∈ R.

4.7. Remark φ(x) = x, x ∈ R, is a solution to (4.58). Set x = eu , y = ev ,


and f (t) = φ(et ), t ∈ R. Then we get f (u + v) = f (u)f (v), u, v ∈ R. From
Exercise (4.6) we obtain that f (t) = at , for any t ∈ R. Thus φ(x) = f (ln x) =
aln x . If a = eα , φ(x) = xα , for all x > 0.
4.1 Solutions 41

4.8. Note the null function is a solution to (4.59). Consider the auxiliary
function
et − e−t
f : R → R \ {−1, 1}, f (t) = t .
e + e−t
It easy to check that function f is one-to-one and onto. Then for any x, y ∈
R \ {−1, 1} there exist u, v ∈ R such that x = f (u) and y = f (v). Moreover,
1 + xy = 1 + f (u)f (v), with 1 + xy ̸= 0.
Consider the function g : R → R defined by g = φ ◦ f. Then (φ ◦ f )(u) +
(φ ◦ f )(v) = (φ ◦ f )(u + v). Since this is precisely the functional equation, we
write (φ ◦ f )(t) = ct, for a constant c. Now we conclude that
c 1+x
φ(x) = cf −1 (x) = ln .
2 1−x

4.9. Suppose a function f : R → R satisfies the first relation. Then

f (xy + x + y) = f (xy) + f (x + y) = f (xy) + f (x) + f (y), x, y ∈ R,

that is, f satisfies the second relation.


Now suppose that a function f : R → R satisfies the second relation. Set
y = u + v + uv. Then

f (x + u + v + xu + xv + uv + xuv)
= f (x) + f (u + v + uv) + f (xu + xv + xuv)

or
f (x + u + v + xu + xv + uv + xuv)
(4.1)
= f (x) + f (u) + f (v) + f (uv) + f (xu + xv + xuv).

In (4.1) we interchange x by u and get

f (x + u + v + xu + xv + uv + xuv)
(4.2)
= f (x) + f (u) + f (v) + f (xv) + f (xu + uv + xuv).

From (4.1) and (4.2) we can write

f (uv) + f (xu + xv + xuv) = f (xv) + f (xu + uv + xuv). (4.3)

In (4.3) we take x = 1 and it follows

f (uv) + f (u + v + uv) = f (v) + f (u + 2uv)

or
f (uv) + f (u) + f (v) + f (uv) = f (v) + f (u + 2uv).
42 4 Limits and Continuity

From here

f (u) + 2f (uv) = f (u + 2uv). (4.4)

Setting u = 0 in (4.4), we get f (0) = 3f (0), so

f (0) = 0. (4.5)

Setting v = −1 in (4.4), we get f (−u) = f (u) + 2f (−u), and

f (−u) = −f (u). (4.6)

Setting v = −1/2 in (4.4), we get f (0) = f (u) + 2f (−u/2) . From (4.5) and
(4.6), we have

f (2u) = 2f (u). (4.7)

From (4.7) and (4.4), we have f (u + 2uv) = f (u) + f (2uv), and by the
substitution 2v = t, it follows

f (u + ut) = f (u) + f (ut). (4.8)

By (4.5), equality f (x + y) = f (x) + f (y) holds for x = 0. Suppose x ̸= 0.


Then by (4.8), we have
( y) ( y)
f (x + y) = f x + x · = f (x) + f x · = f (x) + f (y).
x x

4.10. The null function is a solution to (4.60). Setting x = y = 0, yields


f (0) = 0. For x = y, we obtain f 2 (2x) = 4f 2 (x) and then, since f (x) ≥ 0,
f (2x) = 2f (x). One idea is to show that f (nx) = nf (x), for all n ∈ N∗ , and
n ≥ 1. Assume that f (kx) = kf (x), for all k = 1, 2, . . . , n. Take y = nx in
(4.62) to get f 2 ((n + 1)x) − f 2 ((n − 1)x) = 4f (nx)f (x), and f 2 ((n + 1)x) =
((n − 1)2 + 4n)f 2 (x). Hence f ((n + 1)x) = (n + 1)f (x) and the proof by
induction is completed.
We follow the way in the Cauchy functional equation, Exercise 4.3, to
show that p, q are positive integers, then q · f (p/q) = f (p) = pf (1), so
f (p/q) = (p/q)f (1) and thus f (r) = rf (1), for all r ∈ Q, r ≥ 0.
Take x = 0 in (4.62) to get f 2 (y) − f 2 (−y) = 0, for all y ∈ R, that is
f (y)−f (−y) = 0, for y ∈ R. Thus f is an even function. Then f (r) = |r|f (1),
for r ∈ Q.
Now we show that f (x) = |x| · f (1), for all x ∈ R. Set an arbitrary real x
and set (rn ) a sequence of rational numbers such that limn→∞ rn = x. Since
rn ∈ Q, for all n, we have f (rn ) = |rn |f (1), for all n ∈ N∗ . Function f is
continuous, so we pass to the limit in the last equality to get f (x) = |x|f (1).
4.1 Solutions 43

Note that a = f (1) ≥ 0, therefore the set of solutions contains exactly the
functions f (x) = a|x|, for some nonnegative a.

4.11. Define g(x) = f (x) − f (tx), for all x ∈ R. Remark that g is continuous
at 0 and g(0) = 0. We have x2 = f (x)−f (tx)−(f (tx)−f (t2 x)) = g(x)−g(tx).
From

g(x) − g(tx) = x2 , g(tx) − g(t2 x) = t2 x2 , ...,


g(tn−1 x) − g(tn x) = t2(n−1) x2 ,

we have g(x) − g(tn x) = x2 (1 + t2 + · · · + t2(n−1) ) = x2 (1 − t2n )(1 − t2 ). We


pass to the limit as n → ∞ and since t ∈ ]0, 1[ , it follows

g(x) − g(0) = x2 /(1 − t2 ) ⇐⇒ f (x) − f (tx) = x2 /(1 − t2 ).

From
x2 t2 x2
f (x) − f (tx) = , f (tx) − f (t 2
x) = , ...,
1 − t2 1 − t2
t2(n−1) x2
f (tn−1 x) − f (tn x) = ,
1 − t2
we have
x2 x2
f (x) − f (tn x) = (1 + t2
+ · · · + t 2(n−1)
) = (1 − t2n ).
1 − t2 (1 − t2 )2

We pass to the limit as n → ∞ and since t ∈ ]0, 1[ , it follows f (x) − f (0) =


x2 /(1 − t2 )2 , for every x ∈ R. So

f (x) = k + x2 /(1 − t2 )2 , x ∈ R.

Conversely, each function of the form f (x) = k + x2 /(1 − t2 )2 , x ∈ R is a


solution of the Exercise.

4.12. Exists m > 0 such that |f (x)| ≤ m, for any x ∈ R. Consider the
auxiliary function g(x) = f (x) − x, for x ∈ R. Then g is continuous. From
g(m) = f (m) − m ≤ 0, g(−m) = f (−m) + m ≥ 0, and the continuity of g, it
follows that there exists a ξ ∈ R such that 0 = g(ξ) = f (ξ) − ξ.

4.13. The ”only if” part is clear. Now suppose that (4.61) is true. Then we
have a sequence in a compact interval. It has at least a limit point. Suppose
that the sequence (xn ) does not converge. Then it has at least two limit
points. Choose p, q two limit points with p < q. There must be points from
the interval ]p, q[ in the sequence. There is an x ∈ ]p, q[ such that f (x) ̸= x.
Set ε = |f (x) − x|/2. Then from the continuity of f we get that for some
44 4 Limits and Continuity

δ > 0 for all y ∈ ]x − δ, x + δ [ , one has |f (y) − y| > ε. On the other hand for
n large enough |xn+1 − xn | < 2δ and |f (xn ) − xn | = |xn+1 − xn | < ε. So the
sequence cannot come into the interval ]x − δ, x + δ[, but also cannot jump
over this interval. Then all limit points have to be at most x−δ (contradicting
that q is a limit point), or at least x+δ (contradicting that p is a limit point).

4.14. Consider g(x) = x3 +x, for all x ∈ R. Function g is a third degree poly-
nomial, so it is continuous and onto. Moreover, it is strictly increasing. Then
the its inverse g −1 is strictly increasing and continuous on R, by Theorem
2.20.
Suppose that f is a solution to the exercise. Then

f (g(x)) ≤ x ≤ g(f (x)), for all x ∈ R.

Applying g −1 to the right hand side inequality, we have

g −1 (x) ≤ f (x), for all x ∈ R. (4.9)

Take y = g(x). Then from the left hand side inequality, we have

f (y) ≤ g −1 (y), for all y ∈ R. (4.10)

From (4.9) and (4.10) we conclude that f (x) = g −1 (x), x ∈ R. Easy to see
that f satisfies the assumption. For, take x ∈ R, y ∈ f (x), f −1 (y) = x, and
g −1 (x) = y. Then g(y) = x and

f (g(x)) ≤ x ≤ g(f (x)) ⇐⇒ x ≤ x ≤ x, for all x ∈ R.

4.15. Since f is continuous, f ([a, b]) is a compact set. Thus the sequence
(xn ) is included in a compact set and thus it is bounded.
Function f being increasing, the sequence (xn ) is monotone. So we have
established that the sequence (xn ) converges, let x∗ ∈ [a, b] be its limit. From
the estimates 0 ≤ |f (xn ) − x∗ | = |xn+1 − x∗ | → 0 as n → ∞, we conclude
that f (x∗ ) = x∗ .

4.16. (a) Yes. Indeed, let A = {x ∈ [0, 1] | f (x) > x}. If f (0) = 0, then x = 0
supplies the answer. If not, A in nonempty ( 0 ∈ A ) and bounded, so it has a
supremum, say a. Then b = f (a). There are two cases.
I case: a < b. Then, since f is monotonic and a is a supremum, we get
b = f (a) ≤ f ((a + b)/2) ≤ (a + b)/2, which contradicts a < b.
II case: b < a. Then we get b = f (a) ≥ f ((a + b)2) > (a + b)2, a contra-
diction. Therefore we must have a = b.
(b) No. Here is an example
4.1 Solutions 45
{
1 − x/2, x ≤ 1/2;
f (x) =
1/2 − x/2, x > 1/2.

4.17. Let ]x − α, x + α ] ⊂ [0, 1] be an arbitrary nonempty open interval. The


function is not monotone in the intervals [x − α, x] and [x, x + α], thus there
exist some real numbers x − α ≤ p < q ≤ x and x ≤ r < s ≤ x + α so that
f (p) > f (q) and f (r) < f (s).
By Theorem 2.10, f has a global minimum in the interval [p, s]. The values
f (p) and f (q) are not the minimum, because they are greater than f (q) and
f (s), respectively. Thus the minimum is in the interior of the interval, i.e., it
is a local minimum. So each nonempty open interval ]x − α, x + α ] ⊂ [0, 1]
contains at least one local minimum.

4.18. Suppose that at least one such function exists. Then it is one-to-one
since

f (x) = f (y) =⇒ f (f (x)) = f (f (y)) =⇒ −x = −y =⇒ x = y.

Since f is one-to-one and continuous, it is strictly monotonic. Then f ◦ f is


strictly increasing. But this is impossible since (f ◦ f )(x) = −x, x ∈ R. Thus,
there is no continuous function satisfying (4.62).

4.19. Note that we have a solution, namely the function f (x) = x, x ∈ R.


We show that there is no other solution.
Suppose there is a solution g : R → R and a number a ∈ R such that
g(a) ̸= a.
If g(a) < a, then since g is strictly increasing, g(g(a)) < g(a). By as-
sumption g(g(a)) = a, so we get a < g(a), contradiction.
If g(a) > a, then since g is strictly increasing, g(g(a)) > g(a). By as-
sumption g(g(a)) = a, so we get a > g(a), contradiction. Now we conclude
the only one solution is f (x) = x, x ∈ R.

4.20. Suppose function f is a solution. Since f is strictly increasing, 0 ≤


f (0) < f (1) < f (2) = 2, from where f (0) = 0 and f (1) = 1. Similarly we
have

f (4) = f (2 · 2) = f (2) · f (2) = 4,


2 < 3 < 4 =⇒ 2 = f (2) < f (3) < f (4) = 4 =⇒ f (3) = 3,
f (6) = f (2 · 3) = f (2) · f (3) = 6,
4 < 5 < 6 =⇒ 4 = f (4) < f (5) < f (6) = 6 =⇒ f (5) = 5.

We prove by induction that f (n) = n, for all n ∈ N∗ .


46 4 Limits and Continuity

Let p1 < p2 < . . . be the set of all primes. It is obvious that f (2k ) = 2k ,
for all k ∈ N∗ , f (3k ) = 3k , for all k ∈ N∗ , and f (2k 3m ) = 2k 3m , k, mN∗ .
mn−1 mn−1
Suppose that f (pm 1 m2 m1 m2
1 p2 . . . pn−1 ) = p1 p2 . . . pn−1 , for all m1 , m2 ,

. . . , mn−1 ∈ N , n ≥ 4. Then pn − 1 are pn + 1 are composed. Their prime
decomposition contain primes from {p1 , p2 , . . . , pn−1 } at nonnegative powers.
Then pn − 1 < pn < pn + 1 implies pn − 1 = f (pn − 1) < f (pn ) < f (pn + 1) =
pn +1. From here one has f (pn ) = pn . Immediately one can see that f (n) = n
for all natural n such that their prime decomposition contain primes from
{p1 , p2 , . . . , pn−1 , pn } at nonnegative powers. Thus f (n) = n, for all natural
n.

4.21. Take an arbitrary but fixed x ∈ R \ Q and two rational sequences (un )
and (vn ) such that un → x, vn → x, and un < x < vn , for all n ∈ N∗ . We
show that f (x) = g(x).
Suppose that function g is increasing. Then g(un ) ≤ g(x) ≤ g(vn ),
f (un ) = g(un ) and f (vn ) = g(vn ). Thus

f (un ) ≤ g(x) ≤ f (vn ). (4.11)

Passing to the limit in (4.11) we get that f (x) = g(x).

4.22. From Exercise 4.3 we have that f (p) = p · f (1), for all p ∈ Q. Con-
sider x an arbitrary irrational number, two rational sequences (an ) and
(bn ) converging to x and satisfying an < x and x < bn , for all n. Then
f (1)an = f (an ) ≤ f (x) ≤ f (bn ) = f (1)bn . Passing to the limit, we get
f (1)x = f (x) ≤ f (x) ≤ f (x) = f (1)x. Thus f (x) = f (1)x, x ∈ R. So,
f (x) = ax, for all x ∈ R.

4.23. From |f (x) − f (y)| < |x − y| for all x, y ∈ [a, b], we have that f is
continuous on [a, b]. Inspired by recurrence we define g(x) = (x + f (x))/2,
x ∈ [a, b]. It is easy to see that g([a, b]) ⊂ [a, b]. Moreover, g is increasing.
So the sequence (an ) is bounded and monotonic. Therefore it is convergent.
Let a0 be its limit. Since f is continuous we pass to the limit in an+1 =
(an + f (an ))/2 and find that f (a0 ) = a0 .

4.24. We may assume that a ≥ b ≥ c. Then

a − c ≥ b − c ≥ 0, a − b ≥ 0 and f (a) ≥ f (b)


=⇒ f (a)(a − b)(a − c) ≥ f (b)(b − c)(a − b)
=⇒ f (a)(a − b)(a − c) + f (b)(b − a)(b − c) ≥ 0.

Also, f (c)(c−a)(c−b) ≥ 0. Summing up these two inequalities, we get (4.65).

4.25. First approach. We introduce the function


4.1 Solutions 47

f (y) = ⌊y⌋ + ⌊y + 1/n⌋ + · · · + ⌊y + n − 1/n⌋ − ⌊ny⌋ .

Function f is defined on R. Moreover, it is periodic, since f (y) = f (y + 1/n)


for all y ∈ R. Then it is enough to consider the case x = k + α, where k is
the integer part of x, while 0 ≤ α < 1/n. Thus f (x) = 0 and equality (4.66)
is established.
Second approach. Take y = n in Exercise 1.19.

4.26. There is a positive p such that f (x + p) = f (x), for all x ∈ R. Then


on [0, p] function f is bounded by a constant, let it be, m ≥ 0. It follows
that function f is bounded on R by the same constant m ≥ 0. Now we recall
Exercise 4.12 and thus this exercise is solved.

4.27. By induction we get that f (x) = f (x + m/n), for all x ∈ Q and m, n ∈


N∗ . Taking successively x = 0 and x = −m/n, it follows that f (±m/n) =
f (0), for all m, n ∈ N∗ . It means that f (x) = f (0), for all x ∈ Q. It is enough
to consider an arbitrary real x0 and a sequence of rationals converging to it.

4.28. (a) First examine the case a = 0. In this case (4.67)


√ is a second degree
equation in f (x), having two
√ real roots, namely (2 ± 2)/4. Only one root
is acceptable, that is (2 + 2)/4. For the other we have a negative √ number
under square root. So, we get the constant function f (x) = (2 + 2)/4, for
all x ∈ R. This function is obviously periodic (of any period) as a constant
function.
Now assume that a ̸= 0. Suppose there exists f a solution. Note that
1/2 ≤ f (x) ≤ 1, for all x ∈ R. Define g(x) = f (x) − 1/2, for all x ∈ R. Then
√ √
g(x + a) = 1/4 − g 2 (x) and g(x + 2a) = 1/4 − g 2 (x + a) = g(x).

So, f (x + 2a) = f (x), for all x ∈ R.


(b) We supply three examples

f (x) = (1/2) (|sin πx/2| + 1) , x ∈ R;


{
1/2, 2n ≤ x < 2n + 1,
f (x) = n ∈ Z, x ∈ R;
1, 2n + 1 ≤ x < 2n + 2

 x 1
 + − n, 2n ≤ x ≤ 2n + 1,
f (x) = 12 √
2
( ) n ∈ Z, x ∈ R.
 2
 + x − n − x − n , 2n + 1 ≤ x ≤ 2n + 2
2 2 2

4.29. First approach. The set J = f (I) is an interval and f is strictly mono-
tonic, by Corollary 4.9. Suppose f is strictly increasing. Take u, v ∈ J arbi-
trary, but fixed, such that u < v; Then let x = f −1 (u) and y = f −1 (v) be
48 4 Limits and Continuity

their counter images and let λ be a real between them. Since f −1 is strictly
monotonic, we have x < λ < y. Then z = f (λ) ∈ ] u, v [ and thus f −1 (z) = λ.
Second approach. By Corollary 4.9, f is strictly monotonic, while by Theorem
2.20, the conclusion follows.

4.30. From (4.30) we have

(xn − x1 )f (xk ) ≤ (xk − x1 )f (xn ) + (xn − xk )f (x1 ), k = 1, 2, . . . , n.

From here for k = 1, . . . , n − 1, we have

(xk+1 − xk )(f (xk ) − f (xk+1 ))


(x2k+1 − x2k )(f (xn ) − f (x1 )) + 2(xn f (x1 ) − x1 f (xn ))(xn − x1 )
≤ .
xn − x1
Let A be the right-hand side of the inequality in the statement of Exercise
4.30. Then adding side by side the previous inequality we get
A ≤ (xn − x1 )(f (x1 ) + f (xn )).

4.31. For k = 0 function f has a discontinuity of the second kind at 0. By


Corollary 7.22, f is not of finite variation. For k = 1, similar to Example 7.1,
consider the partition P = ∑{0, 1/(nπ+π/2), 1/(nπ), . . . , 1/(π+π/2), 1/π, 2/π}
n
and note that V (f, P ) = 2 m=1 1/(mπ +π/2)+2/π. This sum tend to ∞ as
n → ∞. So the function is not of finite variation. For k ≥ 2, f has a bounded
derivative on the whole interval, thus is Lipschitz. By (b) in Remarks 7.1, f
is of bounded variation.

4.32. The partial sum is sn = nx/(1 + n2 x2 ). Since for each x ∈ [0, 1]


limn→∞ sn (x) = 0, the series converges pointwise to the null function (which
is continuous). Since for xn = 1/n we have |sn (xn ) − 0| = 1/2, the series
converges non uniformly.

4.33. It is clear that each term of the series is a discontinuous function. The
partial sum is sn = f /(n + 1). Since |sn (x) − 0| ≤ |f (x)|/(n + 1), for each
x ∈ R, we conclude that the series converges uniformly to the null function.
5
Differential Calculus on R

5.1. We write dn as a convex combination of the form

f (βn ) − f (αn ) f (βn ) − f (0) βn f (0) − f (αn ) −αn


= · + · ,
βn − αn βn βn − αn −αn βn − αn
βn −αn
λ1 = , λ2 = , λ1 , λ2 > 0, λ1 + λ2 = 1.
βn − αn βn − αn
It follows
{ }
f (βn ) − f (0) f (0) − f (αn ) f (βn ) − f (αn )
min , ≤
βn −αn βn − αn
{ }
f (βn ) − f (0) f (0) − f (αn )
≤ max , .
βn −αn

The assumptions guarantee that

lim (f (βn ) − f (0))/βn = lim (f (0) − f (αn ))/(−αn ) = f ′ (0).


n→∞ n→∞

Now invoking Theorem 1.18 from page 81, we get the conclusion.
(b) Consider the identity
( )
f (βn ) − f (αn ) f (0) − f (αn ) βn f (βn ) − f (0) f (0) − f (αn )
− = − .
βn − αn −αn βn − αn βn −αn

The sequence (βn /(βn − αn ))n is bounded and since the difference in the
parenthesis tends to 0, the conclusion follows.
(c) Invoking the mean value Theorem 2.3, we write

dn = (f (βn ) − f (αn ))/(βn − αn ) = f ′ (γn ),


50 5 Differential Calculus on R

for a sequence (γn ) satisfying αn < γn < βn . Then γn → 0 and the conclusion
follows since f ′ is continuous.
(d) Consider the function given in (b) of Examples 1.1 and the sequences
(αn ) and (βn ) defined as βn = 2/((4n + 1)π), respectively αn = 1/(2nπ).

5.2. Suppose there exists a point u ∈ ] a, b[ so that g(u) = 0. Since g is


nonconstant, there exists v ∈ ]a, b[ so that g(v) ̸= 0. We may suppose that
u < v. Define w = sup{x ∈ [u, v] | g(x) = 0}. Since g is continuous on ]a, b[ ,
g(w) = 0 and g(x) ̸= 0 for every x ∈ ] w, v [ . On the interval ]w, v] define
h = f /g. Then h′ (x) = 0 for all x ∈ ]w, v [ . By (b) of Theorem 2.4 and
continuity of g we have that h(x) = f (v)/g(v) and f (x) = g(x)f (v)/g(v) for
all x ∈ ] w, v] and

f (v)
f (w) = lim f (x) = lim g(x) = 0.
x↓w x↓w g(v)

Thus we get that f (w) + g(w) = 0, a contradiction. Therefore g has no zero


on ]a, b[ and h is defined and constant on ]a, b[ .

5.3. The inequality is equivalent with ln a/a ≥ ln x/x. So, we are led to
consider the function f (x) = ln x/x, for x > 0. This function has a unique
maximum at x = e. Thus the answer follows for a = e.

5.4. Define g(x) = f (x) · e−x on [0, 1]. Since g(0) = g(1), we find a point
ξ ∈ ]0, 1[ so that g ′ (ξ) = 0. Then 0 = g ′ (ξ) = (f ′ (ξ) − f (ξ))e−ξ , that is
f (ξ) = f ′ (ξ).

5.5. It is clear that all the terms are positive if x1 is so. It is also clear that
if x1 = 0, all the terms are null.
Define the sequence of polynomials

P0 (x) = x, Pn (x) = Pn−1 (x) (Pn−1 + 1/n) , n = 1, 2, . . . .

We have xn = Pn−1 (x1 ). Also Pn (0) = 0, Pn (1) > 1 for n = 1, 2, 3, . . . .


Since Pn (x) has nonnegative coefficients, it is strictly increasing in the interval
I = [0, 1]. Hence one finds (unique) solutions an and bn to

Pn (an ) = 1 − 1/n, Pn (bn ) = 1, n = 1, 2, 3, . . . .

Since

Pn+1 (an ) = Pn (an ) (Pn (an ) + 1/n) = 1 − 1/n < 1 − 1/(n + 1),

we have an < an+1 . Similarly

Pn+1 (bn ) = Pn (bn ) (Pn (bn ) + 1/n) = 1 + 1/n > 1 + 1/(n + 1),
5.1 Solutions 51

so bn > bn+1 . Thus (an ) is increasing, (bn ) is decreasing, and an < bn , for all
n > 1. Then there exists at least one point x1 satisfying an < x1 < bn , for all
n > 1. Hence 1 − 1/n < Pn (x1 ) < 1, for all n > 1. But Pn (x1 ) = xn+1 . So
xn+1 < 1, for all n > 1. Also xn > 1 − 1/n implies xn+1 = xn (xn + 1/n) >
xn . So, sequence (xn ) satisfies all the required conditions.
It remained to consider uniqueness. Pick an x1 satisfying the given con-
ditions. Then 1 − 1/n < Pn (x1 ) < 1, for all n > 1. So we have for all n > 1
that an < x1 < bn .
The uniqueness of x1 will follow by showing that bn − an → 0, as n → ∞.
Since all the coefficients of Pn (x) are nonnegative, it has increasing derivative.
But Pn (0) = 0 and Pn (bn ) = 1, so for any x ∈ [0, bn ] we have Pn (x) ≤ x/bn .
Then we have 1 − 1/n < an /bn and
n→∞
0 < bn − an ≤ bn − bn (1 − 1/n) = bn /n < 1/n −−−−→ 0.

5.6. The inequality has been already proved geometrically by Exercise 1.51 at
page 39. Here we prove it using Theorem 2.4. We only show that sin x < x,
for every x ∈ ]0, π/2[ . Consider the function f defined by f (x) = x−sin x on
x ∈ [0, π/2[ . Then f (0) = 0 and f ′ (x) = 1−cos x > 0, for every x ∈ ]0, π/2[ .
Hence sin x < x on 0 < x < π/2.

5.7. (a) One might apply directly Theorem 4.1. Instead of it we choose writing
the limit as
( ( 2 ))
xln x ln x
lim = exp lim x − ln ln x .
x→∞ (ln x)x x→∞ x

By Theorem 4.1 we have

lim ln2 x/x = lim 2 ln x/x = lim 2/x = 0.


x→∞ x→∞ x→∞
( )
Therefore limx→∞ ln2 x/x − ln ln x = −∞ and so the limit is equal to 0.
We can try finding the answer by the Mathematica r commands.
f[x_] := x^(Log[x])/(Log[x])^x
Limit[f[x], x -> Infinity]
(b) We write
( )
ln x
lim xx = exp(lim x ln x) = exp lim .
x↓0 x↓0 x↓0 1/x

By Theorem 4.1 we further have that the previous limit is equal to 0. We


conclude that the limit is equal to 1.
We can try finding the answer by the Mathematica r command.
Limit[x^x, x -> 0]
52 5 Differential Calculus on R

(c) Since
( )
lim ln(ex + x)/x = lim 1 + ln(1 + xe−x )/x = 1
x→∞ x→∞

we successively have
(√ √ )
ln(ex + x)
x3 + x2 + x + 1 − x2 + x + 1 ·
3
lim
x→∞ x
(√ √ )
x3 + x2 + x + 1 − x2 + x + 1
3
= lim
x→∞
(√ √ )
1 1 1 1 1
1+ + 2 + 3 − 1+ + 2
3
= lim x
x→∞ x x x x x
(1 + t + t2 + t3 )1/3 − (1 + t + t2 )1/2
= lim
t↓0 t
( )
2
1 + 2t + 3t 1 + 2t 1
= lim √ − √ =− .
t↓0 3 3 (1 + t + t2 + t3 )2 2 1 + t + t2 6

5.8. A more general result will be proved, namely f (t) ≥ 0, for all t ∈ ]0, π].
Suppose f (x) ≤ 0 for all x ∈ [0, t]. We show that in this case f (x) = 0
for all x ∈ [0, t]. Indeed, if f (u) < 0 for some u ∈ ]0, t [ , by the mean value
Theorem of Lagrange there exists v ∈ ]0, u[ so that

f ′ (v) = (f (u) − f (0))/u < 0.

We apply the mean value Theorem of Lagrange to the function af ′ to get

(a · f ′ )′ (w) = (a(v)f ′ (v) − a(0)f ′ (0))/v < 0

for some w ∈ ]0, v [ . In this cases (a(w)f ′ (w))′ + f (w) < 0, a contradiction.
Suppose f (p) > 0 for some p ∈ ]0, t [ . Define

ξ1 = sup{x | x ∈ [0, p], f (x) ≤ 0}, ξ2 = inf{x | x ∈ [p, t], f (x) ≤ 0}.

We have 0 ≤ ξ1 < ξ2 < t, f (ξ1 ) = f (ξ2 ) = 0, and f (x) > 0, for all
x ∈ ]ξ1 , ξ2 [ . On the interval ] ξ1 , ξ2 [ define g(x) = a(x)f ′ (x)/f (x). Then

(a(x)f ′ (x))′ g 2 (x)


g ′ (x) = − ≥ 1 − g 2 (x). (5.1)
f (x) a(x)

We note that function g has arbitrary large values in a neighborhood of ξ1


and arbitrary small values in a neighborhood of ξ2 . Indeed, let ε > 0. Let η
be the point of maximum to f on [ξ1 , ξ1 + ε]. By the mean value Theorem of
Lagrange we find a point θ ∈ ]ξ1 , η [ fulfilling
5.1 Solutions 53

f (η) − f (ξ1 ) f (η) f (η)


f ′ (θ) = = ≥ .
η − ξ1 η − ξ1 ε
Then
a(θ)f ′ (θ) f ′ (θ) f ′ (θ) 1
g(θ) = ≥ ≥ ≥ .
f (θ) f (θ) f (η) ε
Similarly, one can show that on [ξ2 − ε, ξ2 ], function g is less than −1/ε.
From (5.1) we have that

g ′ (x)
g ′ (x) ≥ −1 − g 2 (x) =⇒ (arctan g(x))′ = ≥ −1 (5.2)
1 + g 2 (x)

for x ∈ ]ξ1 , ξ2 [ . One the other side, for arbitrary positive δ there are θ1 , θ2 ∈
] ξ1 , ξ2 [ , with ξ1 < ξ2 , so that
π π
arctan g(ξ1 ) > −δ and arctan g(ξ2 ) > − + δ.
2 2
By the mean value Theorem of Lagrange we find a point ξ ∈ ]θ1 , θ2 [ fulfilling

(arctan g)(θ2 ) − (arctan g)(θ1 ) −π + 2δ


(arctan g)′ (ξ) = < .
θ2 − θ1 ξ2 − ξ1

For sufficiently small δ the last fraction is less than −1, contradicting (5.2).
This contradiction shows that the assumption that f (p) > 0 for some p ∈
]0, t [ is false. Now the conclusion follows.

5.9. From the given equality we may write that for y ̸= 0 we have

f (x + y) − f (x − y)
f ′ (x) = , x ∈ R.
2y
The right hand side is differentiable at x, so
( )
′′ f ′ (x + y) − f ′ (x − y) 1 f (x + 2y) − f (x) f (x − 2y) − f (x)
f (x) = = −
2y 2y 2y −2y
f (x + 2y) + f (x − 2y) − 2f (x)
= .
4y 2
Differentiating again we find

f ′ (x + 2y) + f ′ (x − 2y) − 2f ′ (x) 1


f ′′′ (x) = 2
= 2·
4y 4y
( )
f (x + 4y) − f (x) f (x − 4y) − f (x) f (x + 4y) − f (x − 4y)
+ − = 0.
4y (−4y) 4y

Thus f ′′′ (x) = 0, for all x ∈ R. Then


54 5 Differential Calculus on R

x2
f ′′ (x) = f ′′ (0), f ′ (x) = f ′′ (0)x + f ′ (0), f (x) = f ′′ (0) + f ′ (0)x + f (0).
2
We note that each second degree polynomial f (x) = ax2 + bx + c, x ∈ R,
satisfies all the requirements of the exercise. ⊓

5.10. First approach. Setting x = y = 0, we find that f (0)(f (0) − 1) = 0.


So, either f (0) = 0 or f (0) = 1. Assume f (0) = 0 and take into account the
identity f (0)f (x) = f (x), for all x ∈ R. Then f (x) = 0, for all real x, that
is, f is the null function, contrary to our assumption.
We suppose that f (0) = 1. We show that f is differentiable at every
point on the real axis. Let x be an arbitrary point of the real axis. From
f (x + y) = f (x)f (y) we have

lim (f (x + y) − f (x))/y = f (x) lim (f (y) − f (0))/y.


y→0 y→0

The limit in the left hand side there exists and we have

f ′ (x) = f (x)f ′ (0), for all x ∈ R.

Set f ′ (0) = a. Then f ′ (x) = a · f (x), for all x ∈ R. Thus f is indefinite


differentiable at each x ∈ R and

f (n) (x) = an f (x), for all x ∈ R and n ∈ N∗ .

Second approach with an extra assumption of continuity. By Exercise 4.6,


under the assumption of continuity we have that f (x) = ax , a > 0. This
function is differentiable on R. In this case we need not the assumption that
f is differentiable at x = 0.

5.11. Consider g(x) = f (x) − x2 /2. Obviously, g is differentiable and g ′ (0) =


1, g ′ (1) = −1. Then g attains its maximum on a point in int [0, 1]. Let ξ
be this point of maximum, that is, g(ξ) = maxx∈[0,1] g(x), ξ ∈ ]0, 1[ . Then
g ′ (ξ) = 0, that is f ′ (ξ) = ξ.

5.12. Set
g(x) = [f (x) + f ′ (x) + · · · + f (n) (x)]e−x .
From the assumption one gets g(a) = g(b). Then there exists c ∈ ]a, b [ such
that g ′ (c) = 0. Substituting in the last equality g ′ (x) = (f (n+1) (x)−f (x))e−x ,
we finish the proof.

5.13. Consider the convex function [0, ∞ [ ∋ x 7→ f (x) = xm .

5.14. We prove the first inequality. We have that A, B, C ∈ ] 0, π [ and A +


B + C = π. Because the sine function is concave on ] 0, π [ , we may write
5.1 Solutions 55

sin A + sin B + sin C A+B+C 3
≤ sin = .
3 3 2

5.15. Since |e−n sin(nx)| ≤ e−n for every n ∈ N and x ∈ R, the series
converges uniformly and absolutely ∑∞on R. Its sum, denoted by f, is defined
on the whole real axis. The series n=1 ne−n cos(nx) of derivatives converges
uniformly and absolutely on R. Pick up an arbitrary x0 ∈ R and an interval
[a, b] so that x0 ∈ [a, b]. Then by Theorem 7.3, f is differentiable and its
derivative is continuous as a uniform convergent series of continuous functions.
Since x0 is arbitrary on R, the conclusion follows.

5.16. First approach. Function f may be written as


{ {
sin x/x, x ̸= 0, 0, x ̸= 0,
f (x) = f1 (x) = + f2 (x) =
1 x=0 −1 x = 0.

Function f1 is continuous on R, so it has a primitive. Function f2 is not a


Darboux function, so it has no primitive. Hence f has no primitive.
Second approach. We note that function f is discontinuous at x = 0, since
limx→0 f (x) = 1 ̸= 0 = f (0). More precisely, x = 0 is a point of discontinuity
of the first kind. Based on Corollary 10.11, function f has no primitive.
Third approach. We show that f is not a Darboux function. Because
limx→0 f (x) = 1, for all V ∈ V(1) exists ] − δ, δ [ ∈ V(0) such that for
all x ∈ ]0, δ [ , f (x) ∈ V. Choose V = ]1/2, 3/2 [ . So f (x) > 1/2 for every
x ∈ ]0, δ [ . Then we conclude that

f ( ] − δ, δ [ ) = {1/2 < f (x) < 3/2 | x ∈ ]0, δ [ } ∪ {0},

which is not an interval. Hence the function f has no primitive on R.


6
Integral Calculus on R

6.1. We can write



n ∑ n ∑ n ( )
n 1 1 1 k k−1
2 2
= 2
= − .
n +k 1 + (k/n) n 1 + (k/n)2 n n
k=1 k=1 k=1

Consider the function f (x) = 1/(1 + x2 ) on x ∈ [0, 1] and the partition


0 = x0 < x1 < · · · < xn = 1, where xk = k/n, k = 0, 1, 2, . . . , n. Then based
on Theorem 1.4, we have

n ∫ 1
n dx π
lim = = arctan 1 − arctan 0 = . (6.1)
n→∞ n2 + k 2 0 1 + x2 4
k=1

We can approach this exercise by the Mathematica r commands


Limit[Sum[n/(n^2 + i^2), {i, 1, n}], n -> Infinity]
TraditionalForm[%]
(*getting*)
1/2 (\[Pi]+I (log(1+I)-log(1-I))).
With some elementary background on complex functions we write

(1/2)(π + ı ln((1 + ı)/(1 − ı))) = (1/2)(π − π/2) = π/4.

This result agrees with (6.1).

6.2. First we note that arctan(b/a)+arctan(a/b) = sign (ab)π/2. Successively


we have
∫ π/4 ∫ π/2
dx dx
I(a, b) = +
0 a2 cos2 x + b2 sin2 x 2 2 2 2
π/4 a cos x + b sin x
58 6 Integral Calculus on R
∫ 1 ∫ 1 b/a a/b
du du 1 1
= + = arctan t + arctan t
0 a2 + b2 u2 0 b2 + a2 u2 ab ab
( ) 0 0
1 b a π 1
= arctan + arctan = .
ab a b 2 |ab|

We can approach this exercise by the Mathematica r commands


Integrate[1/(a^2 (Cos[x])^2 + b^2 (Sin[x])^2), {x, 0, Pi/2}]
TraditionalForm[%]
(*getting*)
(\[Pi] Sqrt[b^2/a^2])/(2 b^2)

6.3. One has


∫ 1√ ∫ 1 ∫ 1
f (x) f (x)
f (x) dx = √ dx ≥ √ dx = a2/3 .
0 0 f (x) 0 a2/3

6.4. Since all ak > 0, we denote gk (x) = fk (x)/ak . Then for all k,
∫1
g (x) dx = 1. By the arithmetic-geometric inequality (Exercise 1.28 at
0 k
page 35 of the book), we have
∫ √ ∫
1
n
1
g1 (x) + g2 (x) + · · · + gn (x)
g1 (x)g2 (x) · · · gn (x) dx ≤ dx = 1.
0 0 n

Then there exists ξ ∈ [0, 1] so that g1 (ξ)g2 (ξ) · · · gn (ξ) ≤ 1, that is
g1 (ξ)g2 (ξ) · · · gn (ξ) ≤ 1. The last inequality is equivalent to

f1 (ξ)f2 (ξ) · · · fn (ξ) ≤ a1 a2 · · · an .

6.5. For 0 ≤ x < 1, the sequence (xn )n tends monotonically to zero as


n → ∞. Therefore ∫ 1
lim cos(xn ) dx = 1.
n→∞ 0
We evaluate ∫ π
lim cos(xn ) dx.
n→∞ 1
Substitute xn = y. Then
∫ π ∫ πn ∫ n
n 1 1 π d sin y
dy
cos(x ) dx = cos y 1−1/n =
1 n 1 y n 1 y 1−1/n
πn ∫ πn
sin y n−1 sin y dy
= 1−1/n
+ 2
.
ny 1 n 1 y 2−1/n
6.1 Solutions 59

Obviously,
πn
sin y n→∞
−−−−→ 0.
ny 1−1/n 1
For n → ∞, we have
∫ πn ∫ πn
n−1 sin y dy n−1 dy 2(n − 1) n→∞
≤ = (1 − π −n/2 ) −−−−→ 0.
n2 1 y 2−1/n n2 1 y 3/2 n2

Thus ∫ π
lim cos(xn ) dx = 1.
n→∞ 0

6.6. Consider an integer n large enough, at least n > T. Function g(nx) is of


period Tn = T /n. Consider the partition of [0, 1] by {x0 = 0, x1 = Tn , x2 =
2Tn , . . . , xm = mTn }, where m is the largest integer satisfying mTn ≤ 1. We
write
∫ 1 ∑ ∫ xk+1
m−1 ∫ 1
f (x)g(nx) dx = f (x)g(nx) dx + f (x)g(nx) dx.
0 k=0 xk xm

First suppose that g takes only positive values. By (h) in Theorem 1.7, we
write for some ξk ∈ [xk , xk+1 ],
∫ xk+1 ∫ xk+1
f (x)g(nx) dx = f (ξk ) g(nx) dx.
xk xk

Further
∫ xk+1 ∫ nxk+1 ∫ (k+1)T ∫ T
1 1 1
g(nx) dx = g(t) dt = g(t) dt = g(t) dt,
xk n nxk n kT n 0
∫ 1
n→∞
f (x)g(nx) dx −−−−→ 0.
xm

Thus
∫ 1 ∑
m−1 ∫ T
1
f (x)g(nx) dx = f (ξk ) g(t) dt + o(1)
0 n 0
k=0
∫ T ∑
m−1
1
= g(t) dt · f (ξk )(xk+1 − xk ) + o(1)
T 0 k=0
∫ T ∫ 1
n→∞ 1
−−−−→ g(t) dt · f (x) dx.
T 0 0
60 6 Integral Calculus on R

Now consider g is not positive. Then there exists a positive constant c


such that g + c is positive. By the earlier arguments one can write
∫ 1 ∫ T ∫ 1
n→∞ 1
f (x)(g(nx) + c) dx −−−−→ (g(t) + c) dt · f (x) dx.
0 T 0 0
∫1
We subtract from the both sides c · 0
f (x) dx and get the result.

6.7. Since |f ′ (x)| ≤ 1 on [a, b], we have |f (x) − f (ξk )| ≤ |x − ξk |. Therefore


∫ b ∑
n n ∫
∑ xk
f (x) dx − f (ξk )(xk − xk−1 ) ≤ |f (x) − f (ξk )| dx
a k=1 k=1 xk−1
∑n ∫ xk
≤ |x − ξk | dx.
k=1 xk−1

Since
∫ xk
(xk − ξk )2 (ξk − xk−1 )2 (xk − xk−1 )2
|x − ξk | dx = + ≤ ,
xk−1 2 2 2

we have
∫ ∑
n ∑
n
b
(xk − xk−1 )2
f (x) dx − f (ξk )(xk − xk−1 ) ≤
a 2
k=1 k=1

1 ∑
n
xk − xk−1 1
≤ = .
b−a 2 2
k=1

The estimate is sharp since for f (x) = x we get equality.

6.8. Consider n a positive integer large enough, a partition a = x0 < · · · <


xn = b, satisfying xk − xk−1 = xk−1 − xk−2 , for k = 2, 3, . . . , n, and

Mk = sup f (x), mk = inf f (x), for all k = 1, 2, . . . , n.


x∈[xk−1 ,xk ] x∈[xk−1 ,xk ]

Then by Theorem 1.2 one has that



n
lim (Mk − mk )(xk − xk−1 ) = 0.
n→∞
k=1

Thus for n large enough we have



n
(Mk − mk )(xk − xk−1 ) < ε/2 . (6.2)
k=1
6.1 Solutions 61

Define {
mk , x ∈ [xk−1 , xk [ , k = 1, 2, . . . , n
h(x) =
f (b), x = b.
Then 0 ≤ h(x) ≤ f (x) ≤ 1, for every x ∈ [a, b]. By (6.2), we still have
∫ β ∫ b
ε
0≤ (f (x) − h(x))dx ≤ (f (x) − h(x))dx < .
α a 2

Now, we look for a function g : [a, b] → {0, 1} such that on every [α, β] ⊂ [a, b]
one has ∫ β
ε
(h(x) − g(x))dx < . (6.3)
α 2

For it, on every interval [xk , xk+1 ] there exists ηk satisfying


∫ xk+1
ηk − x k = h(x) dx = mk (xk+1 − xk ).
xk

Define 

1, x ∈ [xk , ηk [ ,
g(x) = 0, x ∈ [ηk , xk+1 [ ,


0, x = b.
Then for each k, ∫ ∫
xk+1 xk+1
g(x) dx = h(x) dx. (6.4)
xk xk

We show that this function g satisfies (6.3). If the boundary points of the
interval [α, β] coincide with some xk ′ , then by (6.4) the left-hand side of
(6.3) is zero.
Suppose there exists an interval [xk , xk+1 ] so that it contains [α, β]. Then
∫ β
b−a
(h(x) − g(x))dx ≤ β − α ≤
α n

and (6.3) holds whenever n is large enough.


In the general case (not covered by the previous two cases) we argue as
follows. We write the integral in (6.3) as
∫ β ∫ xk
(h(x) − g(x))dx ≤ (h(x) − g(x))dx
α α


p ∫ xk+m+1 ∫ β
+ (h(x) − g(x))dx + (h(x) − g(x))dx .
m=0 xk+m xk+p+1
62 6 Integral Calculus on R

The sum of integrals is zero since each term is so. The sum of the two extreme
integrals is less then ε/2, whenever n is large enough.

6.9. For k = 0 or k = p, the inequality is trivial.


We show the inequality by induction. If p = 1, the inequality is trivial.
Suppose p = 2 and k = 1, that is

|f ′ (x)| ≤ 2M0 M2 , for all x ∈ R.

Suppose there exists x0 ∈ R so that



|f ′ (x0 )| > 2M0 M2 .

We suppose that f (x0 ) ≥ 0 and f ′ (x0 ) ≥ 0, since if f (x0 ) ≥ 0 and f ′ (x0 ) < 0,
we may consider instead of f, the function R ∋ x 7→ f (2x0 − x) and if
f (x0 ) < 0, we consider the function R ∋ x 7→ −f (x). Thus

f ′ (x0 ) > 2M0 M2 .

For x ≥ x0 , we have
∫ x ∫ x
f ′ (x) ≥ f ′ (x0 ) + f ′′ (u) du ≥ f ′ (x0 ) + (−M2 ) du
x0 x0

= f ′ (x0 ) − M2 (x − x0 ) > 2M0 M2 − M2 (x − x0 )

and
∫ x ∫ x √

f (x) = f (x0 ) + f (u)du > f (x0 ) + ( 2M0 M2 − M2 (x − x0 ))du
x0 x0
√ 1
= f (x0 ) + 2M0 M2 (x − x0 ) − M2 (x − x0 )2
2
√ 1
≥ 2M0 M2 (x − x0 ) − M2 (x − x0 )2 .
2
The
√ point of maximum to the right-hand side is attained at x = x0 +
2M0 M2 /M2 . The √ value of the right-hand side on this point is M0 . Then
we get that f (x0 + 2M0 M2 /M2 ) > M0 , which is a contradiction. Thus the
inequality is proved in this case.
Suppose now that p > 2 and 0 < k < p are integers. Denote
1−k/p
ak = Mk /2k(p−k)/2 M0 Mpk/p .

Then

ak ≤ ak−1 ak+1 , a0 = ap = 1.
By Exercise 3.18 at page 142, we have that max{a0 , a1 , . . . , ap } = max{a0 , ap }
= 1. Then all ak ≤ 1 and thus the exercise is solved.
6.1 Solutions 63

6.10. Note that f (x) = x2 , x ∈ R, is a solution. Thus the set of solutions is


nonempty.
We are looking for other solutions. Setting x = y = 0, we find that
f (0) = 0. For each x ∈ R from f (x + y) − f (x) = f (y) + 2xy, it follows

f (x + y) − f (x) f (y) + 2xy f (y) − f (0)


f ′ (x) = lim = lim = 2x + lim
y→0 y y→0 y y→0 y
= 2x + f ′ (0).

So we are looking for functions f satisfying


∫ x
f (x) = f (x) − f (0) = f ′ (y) dy = x2 + f ′ (0)x.
0

We note for every real a the function f (x) = x2 + ax, x ∈ R satisfies all
the requirements of the exercise.

6.11. The idea is to write the double sum as an integral and then to use
the Cauchy-Buniakovski-Schwarz inequality for integrals, (i) in Theorem 1.7.
Successively we have

n ∑ n ∫ 1
kl
ak al = kak · lal tk+l−2 dt
k+l−1 0
k,l=1 k,l=1
 
∫ 1 ∑ n ∫ 1 (∑
n
)2
=  kak · lal · t k−1+l−1  dt = kak · t k−1
dt.
0 k,l=1 0 k=1

Applying the Cauchy-Buniakovski-Schwarz inequality for integrals, we get


∫ ( )2 (∫ ( ) )2 ( )2
1 ∑
n 1 ∑
n ∑
n
kak · t k−1
dt ≥ kak · t
k−1
dt = ak .
0 k=1 0 k=1 k=1

6.12. We use the rearrangement inequality (Exercise 1.38 at page 38 of the


book).
For each positive integer n we defined the step function
{
fn (0) = f (0),
fn (x) =
inf x∈(k/n,(k+1)/n] f (x), whenever x ∈ ] k/n, (k + 1)/n].

Function fn has all the properties of f, but continuity.


First suppose α is a rational, that is α = p/n, for an integer p. We show
64 6 Integral Calculus on R
∫ 2 ∫ 2
fn (x)fn (x + α) dx ≥ fn (x)fn (x + 1) dx. (6.5)
0 0

Denote ak = inf x∈(k/n,(k+1)/n] f (x). Then



1 ∑
2 2n−1
fn (x)fn (x + α) dx = ak ak+p
0 n
k=0

and (6.5) is equivalent to


2n−1 ∑
2n−1
ak ak+p ≥ ak ak+n . (6.6)
k=0 k=0

System of numbers {ak+p } is obtained from the system {ak+n }. By hypoth-


esis we have that if ai ≤ aj , then ai+n ≥ aj+n . Therefore (6.6) follows from
the rearrangement inequality.
Suppose now that α is arbitrary. We show that
∫ 2 ∫ 2
f (x)f (x + α) dx ≥ f (x)f (x + 1) dx.
0 0

For every positive ε we can find a rational number p/n such that |p/n − α| <
ε. Suppose n is large enough, at least n > 1/ε. Since f is uniformly contin-
uous, for ε → 0, we have
∫ 2 ∫ 2
f (x)f (x + α) dx = f (x)f (x + p/n) dx + o(1)
0 0
∫ 2 ∫ 2
= fn (x)fn (x + p/n) dx + o(1) ≥ fn (x)fn (x + 1) dx + o(1)
0 0
∫ 2
= f (x)f (x + 1) dx + o(1),
0

and the inequality follows.

6.13. It is sufficient to show that


∫ ∞( )
1 1
− dx < ∞, (6.7)
1 u u + u′

since the sum of (6.65) and (6.7) supplies the answer.


We have
1 1 u′ u′
− ′
= ′
≤ 2.
u u+u u(u + u ) u
Then
6.1 Solutions 65
∫ ∞ ( ) ∫ ∞
1 1 u′ dx 1 1
− dx ≤ = − lim .
1 u u + u′ 1 u 2 u(1) x→∞ u(x)
Since the last limit finite (u is increasing), we conclude that (6.7) holds and
the exercise is proved.

6.14. Since the general term of the sequence of partial sums is sn =


2n2 xe−n x , limn→∞ sn = 0, for every x ∈ [0, 1]. So the series converges
2 2

pointwise to the function f (x) = 0, x ∈ [0, 1]. The series does not converges
n→∞
uniformly since for xn = 1/n, we have sn (xn ) = 2n/e −−−−→ ∞. On the
∫1
other side 0 f (x)dx = 0 whereas
∞ ∫
∑ 1 ( )
2x n2 e−n2 x2 − (n − 1)2 e−(n−1) x dx
2 2

n=1 0


(1/e−(n−1) − 1/e−n ) = 1.
2 2
=
n=1

Hence the answer is no.

6.15. We have 3x/(x2 + 5x + 6) = 9/(x + 3) − 6/(x + 2). Also

∑∞
9 3 xn
= =3 (−1)n n , for |x| < 3,
x+3 1 + x/3 n=0
3
∑∞
−6 −3 xn
= = −3 (−1)n n , for |x| < 2.
x+2 1 + x/2 n=0
2

Since both series converge absolutely for |x| < 2, we write

∑∞ ( )
3x 1 1
= 3 (−1) n+1
− xn , |x| < 2.
x2 + 5x + 6 n=0
2n 3n

6.16. Denote arctan x, x ∈ R. Then g ′ (x) = 1/(1 + x2 ) and therefore


∑∞ g(x) n= 2n

g (x) = n=0 (−1) x , |x| < 1. We integrate the last equality from 0 to x
and it follows
∑∞
x2n+1
arctan x = (−1)n , |x| < 1.
n=0
2n + 1
Substituting x = −1 and x = 1 in the right-hand side of the last equality
we get two convergent series. So the series of arctan x converges on |x| ≤ 1.
Squaring the series of arctan x, we get
66 6 Integral Calculus on R

∑∞ ( ) 2n+2
1 1 x
2
(arctan x) = 1 + + ··· + , |x| < 1.
n=0
3 2n + 1 n+1

Easily can be checked that the last identity holds also for x = ±1.

6.17. First we check that the integral converges. We


∫ ∞write xn e−x = xn e−x/2 ·
−x/2 n −x/2 −x/2
e and note that limx→∞ x e = 0 and 0 e dx is convergent.
Then by Theorem 3.7 of Abel we have that our integral converges. By Theorem
3.9, we have
∫ ∞

In = xn e−x dx = −xn e−x|0 + n · In−1 = n · In−1 .
0
∫∞
Since 0 e−x dx = 1, we have In = n!.
One can try using one of the Mathematica r command.
Integrate[x^n*Exp[-x], {x, 0, Infinity},
Assumptions -> n \[Element] Integers && n > 0]

6.18. Since the integrand is not defined at x = 1, we ∫have to take into ac-
1
count several subcases. We show that the integral I1 = 0 x ln x dx/(1 − x2 )2
∫1
diverges. But instead of it we discuss the integral I = 1/e x ln x dx/(1 − x)2 ,
showing its divergence. Define f (x) = −2x ln x = x − 1, for x ∈ I = [1/e, √1].
We have f (1/e) = (3−e)/e > 0, √ f (1) = 0, f is nondecreasing on [1/e, 1/ e],
and f is nonincreasing on [1/ e, 1]. Therefore f is nonnegative on I. Thus
−2x ln x ≥ 1 − x, x ∈ I. Immediately we have
∫ 1 ∫ 1
−2x ln x 1 −2x ln x dx dx
≥ =⇒ ≥ = +∞.
(1 − x) 2 1−x 1/e (1 − x)
2
1/e 1 − x

Thus I diverges and hence I1 diverges, too. We conclude that the initial
integral diverges.

6.19. The
∫ a integrand is not defined at x = 1. Consider 0 < a < 1 and
I(a) = 0 (ln(1/(1 − x)))dx. We get I(a) = (1 − ln(1 − a))(1 − a) + 1. The
integral is equal to lima↑1 I(a) = 1, so it converges.
We can approach this exercise by the Mathematica r command.
Integrate[Log[1/(1 - x)], {x, 0, 1}]
(*getting*) 1

6.20. The integral is improper since at 0 and π/2 the integrand might be
undefined. Consider x = sin u. Then
∫ π/2 ∫ 1
sina−1 u cosb−1 u du = xa−1 (1 − x2 )(b−1)/2 (1 − x2 )−1/2 dx
0 0
6.1 Solutions 67
∫ 1
= t(a−1)/2 (1 − t)b/2−1 (1/2)t−1/2 dt = (1/2)B(a/2, b/2).
0

We can approach this exercise by the Mathematica r command.


Integrate[{Sin[x]}^(a - 1) {Cos[x]}^(b - 1), {x, 0, Pi/2},
Assumptions -> a > 0 && b > 0]
TraditionalForm[%]
(*getting*)
(\[CapitalGamma](a/2) \[CapitalGamma](b/2))
/(2\[CapitalGamma]((a+b)/2))

6.21. For every x ∈ [0, 1] we have limy→0 f (x, y) = 0. For x = y, we have


y→0
f (y, y) −−−−→ ∞. So the convergence to the null function is not uniform in
∫1 ∫1
y. We have limy→0 0 f (x, y)dx = 1/2 whereas 0 limy→0 f (x, y)dx = 0.
7
Differential Calculus on Rn

7.1. If a1 = · · · = an = 0, f is the null functional and its norm is equal to 0.


Suppose that m = a21 + · · · + a2n > 0. It is clear that f is a linear mapping.
Let B be the closed unit ball in Rn . To find the norm of f, we use formula
(7.1). From one side by the Cauchy inequality, Exercise 1.35 in Chapter 1, we
have
v v
u n u n
u∑ u∑ √
∥f ∥ = max |f (x)| ≤ t a2k t x2k ≤ m.
x∈B
k=1 k=1

For the reverse inequality choose √ a particular point in B, namely x0 =


(x1 , x2 , . . . , xn ) by xk = ak / m, k = 1, . . . , n. Clearly, x0 ∈ B and
0 0 0 0

√ √
∥f ∥ ≥ |a1 x01 + · · · + an x0n | = m/ m = m.

Thus ∥f ∥ = m.

7.2. Denote t = x2 + y 2 . Then we have


∂u ∂t ∂u ∂t
= φ′t = φ′t · 2x and = φ′t = φ′t · 2y.
∂x ∂x ∂y ∂y
Then
∂u ∂u
y −x = φ′t · (y(2x) − x(2y)) = 0.
∂x ∂y

7.3. Pick an arbitrary but fixed a ∈ A. Denote L = f ′ (a). We show that


∥L∥ ≤ M. There exists a function r(a, · −a) such that for every x ∈ A, f (x) =
f (a) + L(x − a) + r(a, x − a). We may write r(a, x − a) = ∥x − a∥ω(a, x − a)
such that limx→a ω(a, a − x) = 0 and consider that ω is continuous at zero.
70 7 Differential Calculus on Rn

For an x of norm one there exists η > 0 such that a + tx ∈ A for all t with
|t| < η. Then

f (a + tx) = f (a) + L(tx) + r(a, tx) = f (a) + tL(x) + ∥tx∥ω(a, tx) =⇒


∥tL(x)∥ ≤ ∥f (a + tx) − f (a) − |t|ω(a, tx)∥
≤ ∥f (a + tx) − f (a)∥ + |t| · ∥ω(a, tx)∥ ≤ M ∥tx∥ + |t| · ∥ω(a, tx)∥.

From here we have ∥L(x)∥ ≤ M + ∥ω(a, tx)∥, for all 0 < t < η. Letting t ↓ 0,
by the continuity of ω, we find out ∥L(x)∥ ≤ M. By Theorem 1.3 we write
∥L∥ ≤ M.

7.4. Consider an arbitrary point (x, y) ∈ R2 and define the function g(t) =
f (tx, ty) for t ∈ [0, 1]. Then g is of class C 1 , and

∂f (tx, ty) ∂f (tx, ty)


g ′ (t) = x +y .
∂x ∂y
By Theorem 1.5 at page 318, we have that
∫ 1
f (x, y) = f (x, y) − f (0, 0) = g(1) − g(0) = g ′ (t)dt.
0

7.5. If α = 0, the conclusion is clear. Otherwise, pick up an arbitrary but


fixed a ∈ Rn . Since limx→a ∥f (x) − f (a)∥/∥x − a∥ = 0, we have that f is
differentiable at a and df (a) = 0. The conclusion follows by Theorem 2.8.

7.6. We show that f ′ (a) = 0, for all a ∈ Rn . Suppose that this is not the
case, i.e., there is an a ∈ Rn so that f ′ (a) ̸= 0. Then for some h ∈ Rn the
scalar f ′ (a)h is nonzero. Admit f ′ (a)h > 0. Pick a real t and for a certain ξ
on the segment defined by a and a + th it holds

f (a + th) = f (a) + f ′ (a)(th) + (1/2)f ′′ (ξ)(th, th),

Immediately we have that f (a + th) ≤ f (a) + tf ′ (a)(h). Letting t → −∞,


we get a contradiction. If f ′ (a)h < 0, we let t → ∞ and again we get a
contradiction. Hence f ′ (a) = 0 for all a ∈ Rn .

7.7. From the system  ∂z



 = 3x2 − 3y = 0,
∂x

 ∂z = 3y 2 − 3x = 0,
∂y
we find two stationary points M1 = (0, 0) and M2 = (1, 1). Since
7.1 Solutions 71

∂2z 2 ∂2z ∂2z


f ′′ (x, y)(h, h) = 2
h1 + 2 h1 h2 + 2 h22 = 6xh21 − 6h1 h2 + 6yh22 ,
∂x ∂x∂y ∂y
we have that

f ′′ (0, 0)(h, h) = −6h1 h2 and f ′′ (1, 1)(h, h) = 6(h21 − h1 h2 + h22 ).

Thus M1 is not an extremum point since f ′′ (0, 0) is neither positive nor


negative defined, while M2 is a minimum point since f ′′ (1, 1) is positively
defined. We have xmin = 1, ymin = 1, and zmin = −1.
We may use equally well Theorem 1.9 of Sylvester to study if the stationary
points are extreme points or not. For M1 we have
[ ]
0 −3
A= =⇒ A1 = 0, A2 = −9.
−3 0

Thus M1 is not an extreme point.


For M2 we have
[ ]
6 −3
A= =⇒ A1 = 6, A2 = 27.
−3 6

Thus M2 is a point of local minimum.

7.8. The Lagrangian function F is

F (x, y, z, λ1 , λ2 ) = xyz + λ1 (x2 + y 2 + z 2 − 1) + λ2 (x + y + z).

The set of stationary points is given by the system of nonlinear equations


∂F
= yz + 2λ1 x + λ2 = 0, (7.1)
∂x
∂F
= xz + 2λ1 y + λ2 = 0, (7.2)
∂y
∂F
= xy + 2λ1 z + λ2 = 0, (7.3)
∂z
∂F
= x2 + y 2 + z 2 − 1 = 0, (7.4)
∂λ1
∂F
= x + y + z = 0. (7.5)
∂λ2
Add (7.1), (7.2), and (7.3) and take into account (7.5) to get

xy + yz + zx + 3λ2 = 0. (7.6)

From (7.4) and (7.5) we get xy + yz + zx = −1/2. Together with (7.6) we


find that
72 7 Differential Calculus on Rn

λ2 = 1/6. (7.7)
Subtracting (7.2) from (7.1), we have

(x − y)(z − 2λ1 ) = 0. (7.8)

Therefore we have two cases.


(i) Suppose z = 2λ1 . Substitute it in (7.2). We get the equation

xz + yz + λ2 = 0. (7.9)

By (7.7) we have that z = 0 is not a solution to (7.9). Therefore we


multiply (7.5) by z to get

xz + yz + z 2 = 0. (7.10)

√ get that z − λ2 = 0. Thus z1,2 = ±1/ 6.
2
From (7.9) and (7.10) we
(i1 ) Suppose z1 = 1/ 6. Then we have the stationary point
√ √ √ √
M0 = (−2/ 6, 1/ 6, 1/ 6), λ1 = 1/2 6, λ2 = 1/6.

(i2 ) Suppose z1 = −1/ 6. Then we have the stationary point
√ √ √ √
M1 = (2/ 6, −1/ 6, −1/ 6), λ1 = −1/2 6, λ2 = 1/6.

(ii) Suppose x = y. Substituting in (7.4) and (7.5), we get the system


{
2x2 + z 2 = 1,
2x + z = 0.

Then x = ±1/ 6. √
(ii1 ) Suppose x = 1/ 6. Then we have the stationary point
√ √ √ √
M2 = (1/ 6, 1/ 6, −2/ 6), λ1 = 1/2 6, λ2 = 1/6.

(ii2 ) Suppose x = −1/ 6. Then we have the stationary point
√ √ √ √
M3 = (−1/ 6, −1/ 6, 2/ 6), λ1 = −1/2 6, λ2 = 1/6.

We have
∂2F ∂2F 2 ∂2F 2 ∂2F ∂2F
d2 F = dx 2
+ dy + dz + dλ 2
1 + dλ22
∂x2 ∂y 2 ∂z 2 ∂λ21 ∂λ22
∂2F ∂2F ∂2F
+2 dx dy + 2 dy dz + 2 dz dx
∂x∂y ∂y∂z ∂z∂x
7.1 Solutions 73

∂2F ∂2F ∂2F


+2 dx dλ1 + 2 dy dλ1 + 2 dz dλ1
∂x∂λ1 ∂y∂λ1 ∂z∂λ1
∂2F ∂2F ∂2F ∂2F
+2 dx dλ2 + 2 dy dλ2 + 2 dz dλ2 + 2 dλ1 dλ2
∂x∂λ2 ∂y∂λ2 ∂z∂λ2 ∂λ1 ∂λ2
(7.11)

It is clear that
∂2F ∂2F ∂2F
= = = 0.
∂λ1 ∂λ2 ∂λ21 ∂λ22
From (7.65) by differentiation we get

x dx + y dy + z dz = 0 and dx + dy + dz = 0. (7.12)

We have
∂2F ∂2F ∂2F
2
= 2λ1 , = 2λ1 , = 2λ1 ,
∂x ∂y 2 ∂z 2
∂2F ∂2F ∂2F
= z, = x, = y,
∂x∂y ∂y∂z ∂z∂x
∂2F ∂2F ∂2F
= 2x, = 2y, = 2z,
∂x∂λ1 ∂y∂λ1 ∂z∂λ1
∂2F ∂2F ∂2F
= 1, = 1, = 1.
∂x∂λ2 ∂y∂λ2 ∂z∂λ2

Substituting the last partial derivatives and (7.12) in (7.11) we write

d2 F = 2λ1 (dx2 + dy 2 + dz 2 ) + 2zdx dy + 2xdy dz + 2ydz dx. (7.13)

Now we have to determine if the quadratic mapping (7.13) is positive definite,


negative definite, or none at each point Mk , k = 0 . . . , 3.
If y = z ̸= x, then from (7.12) we have that dx = 0 and dz = −dy. Thus

d2 F |M0 or M1 = 2λ1 (2dy 2 ) − 2x dy 2 = 2(2λ1 − x)dy 2


M0 √ √ √
= 2(1/ 6 + 2/ 6)dy 2 = 6 dy 2 > 0 =⇒ M0 is a point of minimum
M1 √ √ √
= − 2(1/ 6 + 2/ 6dy 2 = − 6 dy 2 < 0 =⇒ M1 is a point of maximum.

If x = y ̸= z, then from (7.12) we have that dz = 0 and dy = −dx. Thus

d2 F |M2 or M3 = 2λ1 (2dx2 ) − 2z dx2 = 2(2λ1 − z)dx2


M2 √ √ √
= 2(1/ 6 + 2/ 6)dx2 = 6 dx2 > 0 =⇒ M2 is a point of minimum
M3 √ √ √
= − 2(1/ 6 + 2/ 6dx2 = − 6 dx2 < 0 =⇒ M3 is a point of maximum.

There are other two cases similar to (7.8), namely


74 7 Differential Calculus on Rn

(y − z)(x − 2λ1 ) = 0 and (z − x)(y − 2λ1 ) = 0.


8
Double Integrals, Triple Integrals, and Line
Integrals

8.1. By Theorem 1.5, we have


∫ ∫ 1 (∫ 2 )
x2 ydx dy = x2 ydy dx
D 0 0
∫ 1 (∫ 2 ) ∫ 1
2
= x ydy dx = 2 x2 dx = 2/3.
0 0 0

One can try using the Mathematica r commands


f[x_, y_] := x^2*y
Integrate[Integrate[f[x, y], {y, 0, 2}], {x, 0, 1}]

8.2. By Theorem 1.5, we have


∫ ∫ 1 (∫ 2 ) ∫ 1 ( )
y=2
(x2 + y)dx dy = (x2 + y)dy dx = (x2 y + y 2 /2) y=0
dx
D 0 0 0
∫ 1 ( )
= 2x2 + 2 dx = 8/3.
0

One can try using the Mathematica r commands


f[x_, y_] := x^2 + y
Integrate[Integrate[f[x, y], {y, 0, 2}], {x, 0, 1}]

8.3. See Figure 8.1. The two curves intersect at (0, 0) and (1, 1). Therefore
by Theorem 1.6, we have
∫ ∫ 1 (∫ √x ) ∫ 1( √ )
y= x
(x + y)dx dy = (x + y)dy dx = (xy + y 2 /2) y=x2 dx
D 0 x2 0
76 8 Double Integrals, Triple Integrals, and Line Integrals

(1,1)

(0,0)

Fig. 8.1. Figure for Exercise 8.3 Fig. 8.2. Figure for Exercise 8.4

∫ 1 ( √ )
= x x + x/2 − (x3 + x4 /2) dx = 3/10.
0

One can try using the Mathematica r commands


f[x_, y_] := x + y
Integrate[Integrate[f[x, y], {y, x^2, Sqrt[x]}], {x, 0, 1}]

8.4. See Figure 8.2 and apply Theorem 1.9. We transform the unit closed ball
in a rectangle by x = ρ cos θ and y = ρ sin θ and get

T (D) = {(ρ, θ) | 0 ≤ ρ ≤ 1, 0 ≤ θ ≤ 2π}.

Then
cos θ −ρ sin θ
det T ′ (x, y) = = ρ.
sin θ ρ cos θ
By equality (8.18), we have
∫ ∫ ∫ 2π ∫ 1
π
(x2 + y 2 )dx dy = ρ2 · ρ dρ dθ = dθ · ρ3 dρ = .
D T 0 0 2

One can try using the Mathematica r commands


f[x_, y_] := x^2 + y^2
Integrate[Integrate[f[x, y],
{y, -Sqrt[1 - x^2], Sqrt[1 - x^2]}], {x, -1, 1}]

8.5. Similarly to the previous Exercise we have x = ρ cos θ, y = ρ sin θ, for


0 ≤ ρ ≤ 1, π/6 ≤ θ ≤ π/3. Then
∫ ∫ ∫ π/3 ∫ 1
(x2 + y 2 )dx dy = ρ2 · ρ dρ dθ = dθ · ρ3 dρ = π/24.
D T π/6 0

One can try using the Mathematica r commands


8.1 Solutions 77

f[x_, y_] := x^2 + y^2


Integrate[Integrate[f[x, y], {y, Sqrt[3]*x/3, Sqrt[3]*x}],
{x, 0, 0.5}] + Integrate[Integrate[f[x, y], {y, Sqrt[3]*x/3,
Sqrt[1 - x^2]}], {x, 0.5, Sqrt[3]/2}]

8.6. The boundary curves intersect at (0, 0) and (2p, 2p).


√ u2 · v
Consider x = √
and y = u · v . The new domain is {(u, v) | 0 ≤ u ≤ 2p, 0 ≤ v ≤ 2p}
2 3 3

whereas the determinant of the transformation is


2uv u2
= 3u2 v 2 .
v 2 2uv

Denote I the integral in the statement. Thus


∫ ∫ √
3
2p ∫ √
3
2p
u3 +v 3 u3 2 3
I= e 2 2
3u v du dv = 3 e u du · ev v 2 du
T 0 0
( √ )2
1 3
3
2p 1 ( 2p )2
= eu = e −1 .
3 0 3

8.7. By Theorem 2.5, we have


∫ ∫ (∫ )
1
2 2
x yzdx dy dz = x yzdy dz dx
D 0 [0,2]×[0,1]
∫ 1 (∫ 2 (∫ 1 ) )
2
= x y zdz dy dx
0 0 0
∫ 1 ∫ 2 ∫ 1
1
= x2 dx · ydy · zdz = .
0 0 0 3
r
One can try using the Mathematica commands
f[x_, y_, z_] := x^2*y*z
Integrate[Integrate[Integrate[f[x, y, z], {z, 0, 1}],
{y, 0, 2}], {x, 0, 1}]

8.8. Denote I the integral in (8.44) in the statement. By Theorem 2.5, we


successively have
∫ 1 (∫ 2 (∫ 1 ) )
I= (x2 + y + z)dz dy dx
0 0 0
∫ 1 (∫ 2 )
2 2 z=1
= (x z + yz + z /2) z=0
dy dx
0 0
∫ 1 (∫ 2 ) ∫ 1
2 y=2
= (x + y + 1/2)dy dx = (x2 y + y 2 /2 + y/2) y=0
dx
0 0 0
78 8 Double Integrals, Triple Integrals, and Line Integrals
∫ 1
= (2x2 + 3)dx = 11/3.
0

One can try using the Mathematica r commands


f[x_, y_, z_] := x^2 + y +z
Integrate[Integrate[Integrate[f[x, y, z], {z, 0, 1}],
{y, 0, 2}], {x, 0, 1}]

8.9. One approach based on Theorem 2.6 is the following. We have


∫ ∫ (∫ 1 )
(x · y · z)dx dy dz = x · y · zdz dx dy
D x2 +y 2 ≤1 x2 +y 2

1
= xy(1 − (x2 + y 2 )2 )dx dy
2 x2 +y2 ≤1
∫ ∫ 2π
1 1 3
= ρ (1 − ρ4 )dρ sin θ cos θdθ = 0.
2 0 0

One can try using the Mathematica r commands


f[x_, y_, z_] := x*y*z
Integrate[Integrate[Integrate[f[x, y, z],
{z, x^2 + y^2, 1}], {y, -Sqrt[1 - x^2], Sqrt[1 - x^2]}],
{x, -1, 1}]

8.10. Apply Theorem 2.8. We transform the unit closed ball into a paral-
lelepiped T by

x = ρ cos α sin β, y = ρ sin α sin β, z = ρ cos β,

where ρ ∈ [0, 1], α ∈ [0, 2π], and β ∈ [0, π]. Then

cos α sin β −ρ sin α sin β ρ cos α cos β


det T ′ (x, y) = sin α sin β ρ cos α sin β ρ sin α cos β = −ρ2 sin β.
cos β 0 −ρ sin β

By equality (8.35), we have


∫ ∫
2 2 2
(x + y + z )dx dy dz = ρ4 sin βdρ dα dβ
D T
∫2π ∫ π ∫ 1
= dα sin βdβ ρ4 dρ = 4π/5.
0 0 0

One can try using the Mathematica r commands


f[x_, y_, z_] := x^2 + y^2 + z^2
8.1 Solutions 79

Integrate[Integrate[Integrate[f[x, y, z],
{z, -Sqrt[1 - x^2 - y^2], Sqrt[1 - x^2 - y^2]}],
{y, -Sqrt[1 - x^2], Sqrt[1 - x^2]}], {x, -1, 1}]


8.11. We have to evaluate D
dx dy dz, where D is given as

{(x, y, z) | x2 + y 2 + z 2 ≤ 1}.

Then ∫ ∫ ∫ ∫
2π π 1
dx dy dz = dα · sin βdβ · ρ2 dρ = 4π/3.
D 0 0 0

One can try using the Mathematica r commands


f[x_, y_, z_] := 1
Integrate[Integrate[Integrate[f[x, y, z],
{z, -Sqrt[1 - x^2 - y^2], Sqrt[1 - x^2 - y^2]}],
{y, -Sqrt[1 - x^2], Sqrt[1 - x^2]}], {x, -1, 1}]

8.12. Use the Taylor formula and a given transformation of variables.


∫ ∫ π/2 √
8.13. C f (x, y)ds = 2 0 1 − cos2 t dt = 2.
One can try using the Mathematica r command
Integrate[Sqrt[2 (1 + Cos[t])]*Sqrt[(1 - Cos[t])^2 + (Sin[t])^2],
{t, 0, Pi/2}]

8.14. The curve is continuous differentiable on its domain and the function
is continuous on the image∫ of the curve. Denote
√ by I the
√ integral. Then after
1 3√
substitution we have I = 0 t 2 + t dt = 8 2/15 − 3/5.
2

One can try using the Mathematica r command


Integrate[t^3 *Sqrt[2 + t^2], {t, 0, 1}]

8.15. The curve γ is continuous differentiable. The function

γ ∋ (x, y) 7→ 1/(x3 + y 3 )

is also continuous differentiable along γ. Then


∫ ∫ ∞

π/2
sin t dt −1 u du −2π 3
I=− 3 = 2 = .
0
2 3
a (sin t + cos t) a 0 1 + u3 9a2


8.16. We write I = I1 + I2 + I3 , where Ik = γk y dx + z dy + x dz and γk is
the segment Ak Ak+1 , with A4 = A1 . Then γ1 is parameterized as x = 1 − t,
80 8 Double Integrals, Triple Integrals, and Line Integrals

y = t, z = 0, with t ∈ [0, 1]; γ2 is parameterized as x = 0, y = 1 − t, z = t,


with t ∈ [0, 1]; γ3 is parameterized as x = t, y = 0, z = 1 − t, with t ∈ [0, 1].
Then ∫ ∫ ∫
1 1 1
I= (−t) dt + (−t) dt + (−t) dt = −3/2.
0 0 0


8.17. The area is given by A = 1/2 γ
x dy − y dx, where γ is given as x =
a cos t, y = b cos t, t ∈ [0, 2π]. Then
∫ 2π
1
A= ab(cos2 t + sin2 t) dt = πab.
2 0

One can try using the Mathematica r commands


(* by the double integral *)
f[x_, y_] := 1
Integrate[ Integrate[ f[x, y], {y, -b*Sqrt[1 - x^2/a^2],
b*Sqrt[1 - x^2/a^2]}], {x, -a, a},
Assumptions -> b \[Element] Reals && a \[Element] Reals && a > 0
&& b > 0]
(* or by the line integral*)
x[t_] := a*Cos[t]
y[t_] := b*Sin[t]
Integrate[(x[t]*D[y[t], t] - y[t]*D[x[t], t])*(1/2), {t, 0, 2*Pi}]

8.18. If |a| = |b|, then immediately follows that F (a, a) = π/(2a4 ).


Suppose that |a| ̸= |b|. By Exercise 6.3 in Chapter 6 we showed that
∫ π/2
I(a, b) = dx/(a2 cos2 x + b2 sin2 x) = π/(2|ab|).
0

There are satisfied the requirements of Theorem 5.3. Then taking the partial
derivatives from I(a, b) in respect to a and b we get
∫ π/2 ∫ π/2
π cos2 x dx π sin2 x dx
= , = .
4a3 b 0 (a2 cos2 x + b2 sin2 x)2 4ab3 0 (a2 cos2 x + b2 sin2 x)2

We add the previous equalities getting F (a, b) = π(a2 + b2 )/(4|ab|3 ).


One can try using the Mathematica r commands
f[x_] := 1/(a^2 {Cos[x]}^2 + a^2 {Sin[x]}^2)^2
Integrate[f[x], {x, 0, Pi/2}]
g[x_] := 1/(a^2 {Cos[x]}^2 + b^2 {Sin[x]}^2)^2
Integrate[g[x], {x, 0, Pi/2},
Assumptions -> b \[Element] Reals
&& a \[Element] Reals && a != 0 && b != 0]
8.1 Solutions 81

8.19. We may suppose that a, b > 0. Note that F (1, 1) = 0. There are
satisfied the requirements of Theorem 5.3. Then taking partial derivatives
from F (a, b) in respect to a and b, we get ∂F (a, b)/∂a = π/(a + b) and
∂F (a, b)/∂b = π/(a + b). From

∂F ∂F
dF = da + db,
∂a ∂b
we find that dF = π d(a + b)/(a + b). Then F (a, b) = π ln(a + b) + C, where
C is a constant. Substituting a = b = 1, we get C = −π ln 2. Thus F (a, b) =
π ln((a + b)/2). If we remove the assumption that a, b > 0, then F (a, b) =
π ln((|a| + |b|)/2).
∫b
8.20. Function f is continuous on [0, 1]. We have that f (x) = a xu du, for
all x ∈ [0, 1]. The integrand is continuous on [0, 1] × [a, b], supposing that
a < b. Then we apply Theorem 5.5. Hence
∫ 1 ∫ 1 (∫ b ) ∫ b (∫ 1 ) ∫ b
u du
f (x) dx = x du dx = xu dx du =
0 0 a a 0 a u+1

b+1
= ln .
a+1

One can try using the Mathematica r commands


f[x_] := (x^b - x^a)/Log[x]
f[0] := 0
f[1] := b - a
Integrate[f[x], {x, 0, 1}, Assumptions -> a > 0 && b > 0]
TraditionalForm[%]

8.21. The integrand is continuous, if we define it as b − c for x = 0. Then we


have to estimate
∫ α −ax
e (sin(bx) − sin(cx))
lim dx.
α→∞ 0 x

We have ∫ b
e−ax (sin(bx) − sin(cx))
e−ax cos ux du = .
c x
The function [0, ∞ [ ×[c, b] ∋ (x, u) 7→ e−ax cos ux is continuous. Therefore
∫ α (∫ b ) ∫ b (∫ α ) ∫ b
−ax −ax
e cos ux du dx = e cos ux dx du = g(u) du,
0 c c 0 c

where
82 8 Double Integrals, Triple Integrals, and Line Integrals

a + e−au (u sin(αu) − a cos(αu))


g(u) = .
a2 + u2
The function g tends uniformly to a/(a2 + u2 ) for α → ∞. Therefore
∫ ∞ ∫
e−ax (sin(bx) − sin(cx)) b
a du b−c
dx = = arctan 2 .
0 x c a2+u 2 a + bc

One can try using the Mathematica r commands


f[x_] := Exp[-a*x] (Sin[b*x] - Sin[c*x])/x
Integrate[f[x], {x, 0, Infinity},
Assumptions -> a > 0 && b \[Element] Reals
&& c \[Element] Reals]
http://www.springer.com/978-0-387-78932-3

You might also like